user8's blog

(Papers) IBPS PO Exam Paper - 2015 "Held on: 31-10-2015" ::QUANTITATIVE APTITUDE::

IBPS logo



(Papers) IBPS PO Exam Paper - 2015 "Held on: 31-10-2015"

::QUANTITATIVE APTITUDE::



DIRECTIONS (Qs. 1-5) : In these questions, a question is given followed by information in three statements. You have to consider the information in all the three statements and decide the information in which of the statement(s) is not necessarily required to answer the question and therefore can be dispensed with. Indicate your answer accordingly.

1. How many students from Institute ‘A’ got placement?

I. Number of students studying in Institutes A & B are in the ratio of 3 : 4 respectively.
II. Number of students who got placement from Institute B is 120% of the number of students who got placement from Institute A.
III. 80% of the students studying in Institute B got placement.

(a) None of the statements can be dispensed with
(b) Only I
(c) Only II
(d) Anyone of the three
(e) Question cannot be answered even with the information in all three statements

2. What is the monthly income of Mr. X?

I. Mr. X spends 85% of his income on various items and remaining amount is saved.
II. Monthly saving of Mr. X are ` 4,500/.
III. Out of the total money spent by Mr. X in a month, one-fifth is spent on food and remaining amount of ` 20, 400 on other items.

(a) Only II
(b) Only III
(c) Only either II or III
(d) Question cannot be answered even with the information in all three statements
(e) None of these

3. What is Suchitra’s present age?

I. Suchitra’s present age is double the age of her son.
II. Ratio between present ages of Suchitra and her mother is 2 : 3 respectively.
III. Four years hence the ratio between Suchitra’s age and her son’s age will be 13 : 24 respectively.

(a) Only II
(b) Only III
(c) Either I or II only
(d) Either II or III only
(e) None of these

4. What is Neeta’s share in the profit earned at the end of 2 years in a joint business run by Neeta, Seeta and Geeta?

I. Neeta invested ` 85,000/ to start the business.
II. Seeta and Geeta joined Neeta’s business after six months investing amounts in the ratio of 3 : 5 respectively.
III. Total amount invested by Seeta and Geeta is ` 2.3 lakhs

(a) Only II
(b) Only III
(c) Only either II or III
(d) Information in all three statements is required for answering the question.
(e) Question cannot be answered even with the information in all three statements.

5. What is the labelled price of the article?

I. Cost price of the article is ` 500/.
II. Selling price after offering 5% discount on the labelled price is ` 608/.
III. Profit earned would have been 28% if no discount was offered.

(a) Only I
(b) Only III
(c) Only II & III
(d) Only I and III
(e) Only I and II

DIRECTIONS (Qs. 6-10) : Study the following graph carefully to answer these questions.

6. What is the average price per kg of wheat and rice sold? (rounded off to nearest integer)

(a) 32
(b) 34
(c) 36
(d) 35
(e) 31

7. If cumin is sold at a 10% discount, the quantity sold doubles. What will be the cost of total quantity of cumin sold on discount?

(a) ` 52,600/
(b) ` 28,800/
(c) ` 32,000/
(d) ` 57,600/
(e) None of these

8. Out of the total quantity of sugar sold, half the quantity is sold at the given rate while half the quantity is sold on a price increased by 20%. What is the total cost of entire quantity of sugar sold?

(a) ` 23,100/
(b) ` 23,800/
(c) ` 22,400/
(d) ` 23,400/
(e) None of these

9. If the quantities sold of Groundnut and Cumin are interchanged, what will be the total cost of quantities sold for these two items together?

(a) ` 62,000/
(b) ` 60000/
(c) ` 65,000/
(d) ` 63,000/
(e) None of these

10. If the quantity of corriander sold is increased by 200% and the price per kg. is reduced by 8%, what will be the total cost of corriander sold?

(a) ` 38, 460/
(b) ` 36,480/
(c) ` 38,640/
(d) ` 36,840/
(e) None of these

DIRECTIONS (Qs. 11-15): Study the following information carefully to answer these questions :

For an examination consisting of three subjects-Maths, Physics and Chemistry, 280 students appeared. When the results were declared, 185 students had passed in Maths, 210 had passed in Physics and 222 had passed in Chemistry. All those except 5 students who passed in Maths, passed in Physics. All those except 10 students who passed in Maths, passed in Chemistry. 47 students failed in all the three subjects. 200 students who passed in Physics also passed in Chemistry.

11. How many students passed in Chemistry only ?

(a) 18
(b) 21
(c) 25
(d) 29
(e) None of these

12. How many students passed in all the three subjects ?

(a) 185
(b) 175
(c) 170
(d) 171
(e) Cannot be determined

13. How many students failed in Physics and Maths ?

(a) 65
(b) 18
(c) 58
(d) 47
(e) Cannot be determined

14. How many students passed in Maths but failed in both Physics and Chemistry ?

(a) 5
(b) 10
(c) 15
(d) 1
(e) Cannot be determined

15. A student has to pass in at least two subjects to get promoted. How many students get promoted ?

(a) 180
(b) 213
(c) 200
(d) 185
(e) Cannot be determined

DIRECTIONS (Qs. 16-20): Study the following Table carefully and answer the question given below :

16. How many students have scored the lowest marks in two or more subjects ?

(a) 2
(b) 3
(c) 1
(d) 0
(e) 4

17. Who has scored the highest marks in all the subjects together ?

(a) Deepak
(b) Charu
(c) Anupama
(d) Garima
(e) Bhaskar

18. What is the percentage of Deepak's marks (upto two digits after decimal) in all the subjects together ?

(a) 88.63
(b) 77.38
(c) 67.83
(d) 62.83
(e) 72.83

19. Marks obtained by Charu in Hindi are what percent of marks (upto two digits after decimal) obtained by Anupama in the same subject ?

(a) 75.92
(b) 78.38
(c) 77.29
(d) 75.29
(e) 72.83

20. What are the average marks obtained by all the students together in Science ?

(a) 55.75
(b) 57.5
(c) 60
(d) 59.5
(e) 58

DIRECTIONS (Qs. 21-25): What should come in place of the question mark (?) in the following number series ?

21. 104 109 99 114 94 ?

(a) 69
(b) 124
(c) 120
(d) 78
(e) None of these

22. 980 392 156.8 ? 25.088 10.0352

(a) 65.04
(b) 60.28
(c) 62.72
(d) 63.85
(e) None of these

23. 14 16 35 109 441 ?

(a) 2651
(b) 2205
(c) 2315
(d) 2211
(e) None of these

24. 1331 2197 4913 6859 ? 24389

(a) 12167
(b) 13824
(c) 9261
(d) 15625
(e) None of these

25.. 3600 725 150 35 12 ?

(a) 8
(b) 7.4
(c) 10.5
(d) 10
(e) None of these

DOWNLOAD IBPS PO Question Papers PDF

DOWNLOAD IBPS CLERK Question Papers PDF

DOWNLOAD MORE BANK EXAMS E-BOOKS

Printed Study Material for IBPS PO Exam

IBPS PO (Probationary Officer) Exam Study Materials

IBPS / SBI Special TX: 
General: 

(Papers) IBPS PO Exam Paper - 2015 "Held on: 31-10-2015" ::ENGLISH LANGUAGE::

IBPS logo


(Papers) IBPS PO Exam Paper - 2015 "Held on: 31-10-2015"

::ENGLISH LANGUAGE::


DIRECTIONS (Qs. 1-15): Read the following passage based on an Interview to answer the given questions based on it. Certain words are printed in bold to help you locate them while answering some of the questions.

A spate of farmer sucides linked to harassment by recovery agents employed by Micro Finance Institutions (MFIs) in Andhra Pradesh spurred the state government to bring in regulation to protect consumer interests. But, while the Bill has brought into sharp focus the need for consumer protection, it tries to micro-manage MFI operations and in the process it could scuttle some of the crucial benefits that MFls bring to farmers, says the author of Microfinance India, State of the Sector Report 2010. In an interview he points out that prudent regulation can ensure the original goal of the MFIs–social uplift of the poor. Do you feel the AP Bill to regulate MFIs is well thought out ? Does it ensure fairness to the borrowers and the long-term health of the sector ? The AP bill has brought into sharp focus the need for customer protection in four critical areas First is pricing. Second is tender’s liability — whether the Lender can give too much loan without assessing the customer’s ability to pay. Third is the structure of loan repayment –
whether you can ask money on a weekly basis from people who don’t produce weekly incomes. Fourth is the practices that attend to how you deal with defaults. But the Act should have looked at the positive benefits that institutions could bring in, and where they need to be regulated in the interests of the customers. It should have brought only those features in. Say, you want the recovery practices to be consistent with what the customer can really manage. If the customer is aggrieved and complains that somebody is harassing him, then those complaints should be investigated by the District Rural Development Authority. Instead what the Bill says is that MFIs cannot go to the customer’s premises to ask for recovery and that all transactions will be done in the Panchayat office. With great difficulty, MFIs brought services to the door of people. It is such a relief for the customers not to be spending time out going to banks or Panchayat offices, which could be 10 km away in some cases. A facility which has brought some relief to people is being shut. Moreover, you are practically telling the MFI where it should do business and how it should do it. Social responsibilities were in-built when the MFIs
were first conceived. If MFIs go for profit with loose regulations, how are they different from moneylenders? Even among moneylenders there are very good people who take care of the customer’s circumstance, and there are really bad ones. A large number of the MFIs are good and there are some who are coercive because of the kind of prices and processes they have adopted. But Moneylenders never got this organised. They did not have such a large footprint. An MFI brought in organisations, it mobilized the equity, it brought in commercial funding. It invested in systems. It appointed a large number of people. But some of then exacted a much higher price than they should have. They wanted to break even very fast and greed did take over in some cases. Are the for-profit MFIs the only ones harassing people for recoveries ? Some not-for-profit outfits have also adopted the same kind of recovery methods. That may be because you have to show that you are very efficient in your recovery methods and that your portfoilo is of a very high quality if you want to get commercial funding from a bank. In fact, among for-profits there are many who have sensible recovery practices. Some have fortnightly recovery, some have monthly recovery. So we have differing practices. We just describe a few dominant ones and assume every for-profit MFI operates like that. How can you introduce regulations to ensure social upliftment in a sector that is moving towards for -profit models ? I am not really concerned whether someone wants to make a profit or not. The bottom-line for me is customer protection. The first area is fair practices. Are you telling your customers how the loan is structured ? Are you being transparent about your performance ? There should also be a lender’s lilability attached to what you do. Suppose you lend excessively to a customer without assessing their ability to service the loan, you have to take the hit. Then there’s the question of limiting returns. You can say that an MFI cannot have a return on assets more than X,
a return on equity of more than Y. Then suppose there is a privately promoted MFI, there should be a regulation to ensure the MFI cannot access equity markets till a certain amount of time. MFIs went to markets perhaps because of the need to grow too big too fast. The government thought they were making profit off the poor, and that’s an indirect reasons why they decided to clamp down on MFIs. If you say an MFI won’t go to capital market, then it will keep political compulsions under rein.

1. Which of the following best explains ‘Structure of loan repayment’ in the context of the first question asked to the author ?

(a) Higher interest rate
(b) Payment on weekly basis
(c) Giving loan without assessing ability to pay
(d) Method of dealing with defaults
(e) Total amount of loan

2. The author is of the view that _____

(a) the bill to regulate MFIs is not needed
(b) the bill neglects the interests of the customers
(c) the positive aspects of MFIs should also be considered.
(d) most of the MFIs are not good.
(e) MFIs must be told what and where they should do business

3. One of the distinct positive feature of MFIs is that _____.

(a) they brought services to the door of people
(b) they dealt with defaulters very firmly
(c) they provided adequate customer protection
(d) they are governed by the local people
(e) they have highly flexible repayment plan

4. What is the difference between MFIs and moneylenders ?

(a) There is no difference.
(b) A large number of money lenders are good whereas only a few MFIs are good
(c) Money lenders gave credit at lower rate of interest than that of MFIs
(d) MFIs adopted a structure and put a process in place, which was not the case with moneylenders
(e) Moneylender appointed large number of local people as against more outside people in MFIs

5. Which of the following is positive outcome of the AP Bill to regulate MFIs ?

(a) The banks have started this service in remote areas
(b) It highlighted some areas of customer protection
(c) It highlighted the bad practices being followed by moneylenders
(d) MFIs is invested in systems and broguth in commercial funding.
(e) It will help convert MFIs into small banks

6. The author is recommending ____.

(a) Not-for profit MFIs
(b) For-profit MFIs
(c) Stoppage of commercial funding to MFIs
(d) Customer satisfaction irrespective of ‘Not-for profit’ or ‘for profit’ MFIs
(e) Public sector promoted MFIs

7. Why did MFIs go to the equity markers ?

(a) To repay the loan
(b) To lower interest rate
(c) There were political compulsions
(d) To become a public sector institution
(e) To grow very fast

8. Which of the following has not been indicated as one of the features of air practices for customer protection ?

(a) Providing information about loan structuring.
(b) MFIs should also be held liable for some of their actions
(c) Not to raise money from capital market
(d) MFIs should also inform public about their own performance also
(e) To provide credit as per the rational assessment of their ability to service the loan

9. Which of the following could possibly be most plausible reason for banning recovery by going to customer’s premises ?

(a) To protect the family members
(b) To protect the customer from harassment and coercion
(c) To reduce the undue expenses of MFIs is resulting in lower interest rates.
(d) To account systematically the money recovered in the books of accounts
(e) To keep Panchayat office out of these transactions

DIRECTIONS (Qs. 10-13) : Choose the word which is most nearly the same in meaning as the word/group of words printed in bold.

10. manage

(a) afford
(b) assess
(c) thrust
(d) administer
(e) use

11. exacted

(a) perfected
(b) demanded
(c) estimated
(d) corrected
(e) accurate

12. scuttle

(a) delay
(b) mix
(c) shuffle
(d) destroy
(e) smoothen

13. spurred

(a) agitated
(b) instigated
(c) reflected
(d) disapproved
(e) prompted

DIRECTIONS (Qs. 14-15) : Choose the word or group of words which is most opposite in meaning of the word printed in bold.

14. under rein

(a) under wrap
(b) without target
(c) let loose
(d) no cloud
(e) under cloud

15. coercive

(a) gentle
(b) promoting
(c) progressive
(d) natural
(e) opinionated

DIRECTIONS (Qs. 16-20) : Rearrange are following six sentences (A), (B), (C), (D), (E) and (F) in the proper sequence to form a meaningful paragraph; then answer the questions given below them.

(A) There are a number of item in the atomic energy programme which are being made indigenously.
(B) Given the overall energy situation in India, the use of nuclear power in some measure is inescapable even while thermal and hydro power continue to be the dominant
elements.
(C) However, commercial aspects of exploiting nuclear capabilities, especially for power-generation programmes, have been recently given high priority.
(D) Atomic energy programmes have been subject to severe restrictions for every obvious reason as the Department of Atomic energy is becoming self-reliant in areas in which only a few countrices have such capability.
(E) Even to meet these nuclear power requirements, India critically requires a commercia level power-generation capability, with its commensurate safety and nuclear waste
management arrangements.
(F) Thus, in Indian context energy security is also crucial, perhaps much more than it is for the U.S.A.. because India imports a good part of its crude oil requirements, paying
for it with precious foreign exchange.

16. Which of the following will be the FIFTH sentence after rearrangement ?

(a) (A)
(b) (B)
(c) (C)
(d) (D)
(e) (E)

17. Which of the following will be the THIRD sentence after rearrangement ?

(a) (A)
(b) (B)
(c) (C)
(d) (D)
(e) (E)

18. Which of the following will be the SECOND sentence after rearrangement ?

(a) (A)
(b) (B)
(c) (C)
(d) (D)
(e) (E)

19. Which of the following will be the FIRST sentence after rearrangement ?

(a) (A)
(b) (B)
(c) (C)
(d) (D)
(e) (E)

20. Which of the following will be the FOURTH sentence after rearrangement ?

(a) (A)
(b) (B)
(c) (C)
(d) (D)
(e) (E)

DOWNLOAD IBPS PO Question Papers PDF

DOWNLOAD IBPS CLERK Question Papers PDF

DOWNLOAD MORE BANK EXAMS E-BOOKS

Printed Study Material for IBPS PO Exam

IBPS PO (Probationary Officer) Exam Study Materials

IBPS / SBI Special TX: 
Subject: 
General: 

(Papers) IBPS PO Exam Paper - 2015 "Held on: 31-10-2015" ::COMPUTER KNOWLEDGE::

IBPS logo



(Papers) IBPS PO Exam Paper - 2015 "Held on: 31-10-2015"

::COMPUTER KNOWLEDGE::



1. When sending an e-mail, the … line describe the contents of the message

(a) Subject
(b) To
(c) Contents
(d) CC
(e) None of these

2. All the deleted files go to

(a) Recycle bin
(b) Task bar
(c) Tool bar
(d) My computer
(e) None of these

3. You organise files by storing them in

(a) Archives
(b) Folders
(c) Indexes
(d) Lists
(e) None of these

4. A website address is a unique name that identifies a specific ___ on the web

(a) Web browser
(b) Website
(c) PDA
d) Link
(e) None of these

5. _________ are specially designed computer chips that reside inside o ther devices, such as your car or your electronic thermostat

(a) Servers
(b) Embedded computers
(c) Robotic computers
(d) Mainframes
(e) None of these

6. Which of the following places the common data elements in order from smallest to largest?

(a) Byte, Bit, Character, file, record, field, database
(b) Character, record, field, file, database
(c) Character, field, record, file, database
(d) Bit, byte, character, record, field, file, database
(e) None of these

7. A disk's content that is recorded at the time of manufacture and that cannot be changed or erased by the user is

(a) Memory only
(b) Write only
(c) Read only
(d) Run only
(e) None of these

8. An error in a computer program

(a) Crash
(b) Power Failure
(c) Bug
(d) Virus
(e) None of these

9. Distributed processing involves

(a) solving computer component problems from a different computer
(b) solving computing problems by breaking them into smaller parts that are separately processed by different computers
(c) allowing users to share files on a network
(d) allowing users to access network resources away from the office
(e) None of these

10. The operating system determines the manner in which all of the following occurs except

(a) user creation of a document
(b) user interaction with the processor
(c) printer output
(d) data displayed on the monitor
(e) None of these

DOWNLOAD IBPS PO Question Papers PDF

DOWNLOAD IBPS CLERK Question Papers PDF

DOWNLOAD MORE BANK EXAMS E-BOOKS

Printed Study Material for IBPS PO Exam

IBPS PO (Probationary Officer) Exam Study Materials

IBPS / SBI Special TX: 
General: 

(Papers) IBPS PO Exam Paper - 2015 "Held on: 31-10-2015" ::GENERAL AWARENESS::

IBPS logo


(Papers) IBPS PO Exam Paper - 2015 "Held on: 31-10-2015"

::GENERAL AWARENESS::


1. Which is the highest populated state of India?

(a) Madhya Pradesh
(b) Uttar Pradesh
(c) Bihar
(d) Maharashtra
(e) Haryana

2. Who is the head of the panel formed RBI to look into parameters for urban co-operative banks recently?

(a) Bimal Jalan
(b) R Gandhi
(c) Urijit Patel
(d) YV Reddy
(e) None of these

3. Which banks have been designated as Domestic Systemically Important Banks (D-SIBs)?

(a) ICICI& SBI
(b) PNB & SBI
(c) HDFC & SBI
(d) ICICI & PNB
(e) None of these

4. Bhakti Sharma, who is in news recently associated with which sports?

(a) Swimming
(b) Tennis
(c) Badminton
(d) Volley Ball
(e) None of these

5. Consider the following statements:

A. Yog Guru Baba Ramdev was appointed as state brand ambassador of Haryana Government
B. It was announced by Minister of Sports and Health of Haryana, Anil Vij
C. Haryana Govt took this decision as it want to promote yoga and ayurveda in Haryana State

(a) Only C
(b) Both A and C
(c) Both A and B
(d) All A, B, C
(e) None of these

6. Frank Tyson was a cricketer from which country?

(a) New Zeland
(b) England
(c) West Indies
(d) South Africa
(e) None of these

7. What is the full form of NBFC as used in the financial sector?

(a) New Banking Finance Company
(b) National Banking & Finance Corporation
(c) Non Business Fund Company
(d) New Business Finance & Credit
(e) None of these

8. Which of the following fund transfer mechanisms, can be moved from one bank to another and where transaction is settled instantly without being bunched with any other
transaction?

(a) RTGS
(b) NEFT
(c) TT
(d) EFT
(e) MT

9. Lima is the capital of.

(a) Brazil
(b) Peru
(c) Ecuador
(d) Colombia
(e) None of these

10. Where is Buxa National Park located?

(a) Odisha
(b) West Bengal
(c) Madhya Pradesh
(d) Bihar
(e) Rajasthan

11. English Stephen Constantine has been appointed as the head coach of Indian men's ____________ team.

(a) Hockey
(b) Cricket
(c) Football
(d) Badminton
(e) None of these

12. Banking ombudsmen is appointed by.

(a) SBI
(b) Indian Government
(c) RBI
(d) President
(e) Prime minister

13. Which committee framed the RRB Act?

(a) Narsimham Committee
(b) Ashok Mehta Committee
(c) Sachar Committee
(d) Shah Nawaz Committee
(e) None of these

14. Ravindra Jain who passed recently was a.

(a) Politician
(b) Singer
(c) Author
(d) Music Composer
(e) None of these

15. Base rate is the rate below which no bank can allow their lending to anyone. Who set up this base rate for banks?

(a) Individual Banks Boards (Correct Answer)
(b) Ministry of Commerce
(c) Ministry of Finance
(d) RBI
(e) Interest Rate Commission of India

16. Where is the Head-quarter of Indian Bank?

(a) Chennai
(b) Mumbai
(c) Delhi
(d) Bengaluru
(e) Hyderabad

17. Many a times, we read in newspaper that the RBI has revised certain rates/ratios under LAF. What is full form of LAF?

(a) Legal Adjustment Facility
(b) Liquidity Adjustment Facility
(c) Longterm Achievement Facility
(d) Legal Adjustment Formality
(e) None of these

18. What is KVP lock period?

(a) 36 Months
(b) 48 Months
(c) 24Months
(d) 30 Months
(e) 40 Months

19. Which of the following statement is true about the Competition Commission?

1. The Competition has been established to prevent practices which do not support healthy business competition.
2. The Commission will have five members including the chairman
3. The Commission has to ensure that the financial operation of any business entity does not get concentrated in the hands of few people.

(a) Only 1
(b) Only 2
(c) Only 3
(d) All of them
(e) None of them

20. What is the cost of Delhi National Memorial Cost?

(a) 100 crore
(b) 1000 crore
(c) 500 crore
(d) 250 crore
(e) None of these

DOWNLOAD IBPS PO Question Papers PDF

DOWNLOAD IBPS CLERK Question Papers PDF

DOWNLOAD MORE BANK EXAMS E-BOOKS

Printed Study Material for IBPS PO Exam

IBPS PO (Probationary Officer) Exam Study Materials

IBPS / SBI Special TX: 
General: 

(Papers) IBPS PO Exam Paper - 2015 "Held on: 4-10-2015" ::REASONING ABILITY::

IBPS logo


(Papers) IBPS PO Exam Paper - 2015 "Held on: 4-10-2015"

::REASONING ABILITY::


DIRECTIONS (Qs. 1-5) : Study the following information carefully and answer the questions given below.

P, Q, R, S, T, V and W are seven students of a school. Each of them studies in different standard from Standard IV to Standard X not necessarily in the same order. Each of them has favourite subject from English, Science, History,Geogra-phy, Mathematics, Hindi and Sanskrit not necessarily in the same order. Q studies in VII Standard and does not like either Mathematics or Geography. R likes English and does not study either in V or in IX. T studies in VIII Standard and likes Hindi. The one who likes Science studies in X Standard. S studies in IV Standard. W likes Sanskrit. P does not study in X Standard. The one who likes Geography studies in V Standard.

1. In which standard does W study?

(a) VII
(b) IX
(c) X
(d) Data inadequate
(e) None of these

2. Which subject does P like?

(a) Geography
(b) Mathematics
(c) English
(d) History
(e) None of these

3. Which subject does S like?

(a) History
(b) Geography
(c) Mathematics
(d) Data inadequate
(e) None of these

4. In which standard does P study?

(a) IV
(b) VII
(c) IX
(d) X
(e) None of these

5. Which of the following combinations of student-standardsubject is correct?

(a) T -VIII -Mathematics
(b) W -VII -Sanskrit
(c) Q -VII -Geography
(d) V -X -Science
(e) None of these

DIRECTIONS (Qs. 6-10) : Study the following information carefully and answer the questions given below:

(i) A, B, C, D, E, F, G and H are eight students each having a different height.
(ii) D is shorter than A but taller than G.
(iii) E is taller than H but shorter than C.
(iv) B is shorter than D but taller than F.
(v) C is shorter than G.
(vi) G is not as tall as F.

6. Which of the following is definitely false?

(a) G is shorter than F
(b) C is shorter than F
(c) F is taller than C
(d) B is taller than E
(e) All are true

7. If another student J who is taller than E but shorter than G is added to the group, which of the following will be definitely true?

(a) C and J are of same height
(b) J is shorter than D
(c) J is shorter than H
(d) J is taller than A
(e) None of these

8. Which of the following will definitely be the third from top when the eight students are arranged in descending order of height?

(a) B
(b) F
(c) G
(d) B or G
(e) Cannot be determined

9. How many of them are definitely shorter than F ?

(a) Three
(b) Four
(c) Five
(d) Data inadequate
(e) None of these

10. Which of the following is redundant to answer all the above questions?

(a) (ii) only
(b) (ii) and (iii) only
(c) (iii) and (iv) only
(d) (i) and (v) only
(e) All are necessary to answer the above questions

DIRECTIONS (Qs. 11-15) : Study the following information carefully and answer the given questions :

In a certain code language :
"demand and supply market" is written as "pa ni de re"
"market needs more demand" is written as "de ja ni fe"
"supply demand is related" is written as "le de re ab"
"more related to economics" is written as "ka ha ab ja"

11. What is the code for 'economics' ?

(a) ab
(b) ka
(c) ha
(d) ja
(e) Either 'ka' or 'ha'

12. Which of the following represents 'supply related market' ?

(a) ab ni de
(b) ni re ab
(c) pa ni re
(d) de ab ni
(e) None of these

13. What is the code for 'more' ?

(a) fe
(b) ni
(c) de
(d) ja
(e) Cannot be determined

14. Which of the following may represent "market needs more customers" ?

(a) fe ja ni sa
(b) ja ni de ab
(c) ni ja ka pa
(d) pa ni fe re
(e) le re ni ja

15. What is the code for 'needs' ?

(a) ni
(b) fe
(c) pa
(d) le
(e) None of these

DOWNLOAD IBPS PO Question Papers PDF

DOWNLOAD IBPS CLERK Question Papers PDF

DOWNLOAD MORE BANK EXAMS E-BOOKS

Printed Study Material for IBPS PO Exam

IBPS PO (Probationary Officer) Exam Study Materials

IBPS / SBI Special TX: 
General: 

(Papers) IBPS PO Exam Paper - 2015 "Held on: 4-10-2015" ::QUANTITATIVE APTITUDE::

IBPS logo



(Papers) IBPS PO Exam Paper - 2015 "Held on: 4-10-2015"

::QUANTITATIVE APTITUDE::



DIRECTIONS (Qs. 1-5): Find the missing term.

1. 0.5, 1.5, 5, 8,76,?

(a) 380
(b) 385
(c) 390
(d) 395
(e) None of these

2. 65, 72, 86, 114 ?

(a) 160
(b) 165
(c) 170
(d) 175
(e) None of these

3. 63, 31, 15, 7, 3 ?

(a) 0
(b) 1
(c) 2
(d) 3
(e) None of these

4. 13. 70, 71, 76, ?, 81, 86, 70, 91

(a) 70
(b) 71
(c) 80
(d) 96
(e) None of these

5. 8, 43, 11, 41, ?, 39, 17

(a) 8
(b) 14
(c) 43
(d) 44
(e) None of these

DIRECTIONS (Qs. 6-10): In each of these questions, two equations (I) and (II) are given. You have to solve both the equations and give answer

(a) if x > y
(b) if x < y
(c) if x ³ y
(d) if x £ y
(e) if x = y or relation cannot be established between 'x' and 'y'.

6. I. 8x + y =10
II. 4x + 2y = 13

7. I. (x + 3) (y + 2) = 12
II. 2xy + 4x + 5y = 11

8. I. (3x – 2)/y = (3x + 6)/(y + 16)
II. (x + 2)/(y + 4) = (x + 5)/(Y + 10)

9. I. x2 + 20x + 4 = 50 – 25x
II. y2 – 10y – 24 = 0

10. I. (x2 – 10x + 16)/(x2 – 12x + 24) = 2/3
II. y2 – y – 20 = 0

DIRECTIONS (Qs. 11-15) : Study the given table carefully to answer the following questions.

11. What is the cost of flooring of A?

(a) ` 4000
(b) ` 4600
(c) ` 4800
(d) ` 5000
(e) ` 4400

12. What is the difference between the cost of fencing of C and that of B?

(a) ` 180
(b) ` 120
(c) ` 240
(d) ` 360
(e) ` 480

13. What is the ratio of the cost of flooring to that of fencing of field D?

(a) 4 : 1
(b) 6 : 1
(c) 8 : 1
(d) 9 : 1
(e) 5 : 1

14. The cost of fencing of field E is approximately what percent of the cost of flooring of field C?

(a) 10.5%
(b) 19.46%
(c) 18.71%
(d) 15.36%
(e) 13.82%

15. The cost of fencing of field C is what percent of the cost of fencing of field D?

(a) 87.54%
(b) 67.5%
(c) 72.13%
(d) 54.36%
(e) 46.5%

DIRECTIONS (Qs. 16-23) : Study the following table and pie chart carefully to answer the given questions.

The table shows the ratio of Hindu religion soldiers to soldiers of other religions

16. What is the number of Hindu soldiers in Jat regiment?

(a) 2600
(b) 2700
(c) 3200
(d) 2800
(e) 2350

17. What is the difference between Hindu soldiers in Madras regiment and soldiers of other religions in Bihar regiment?

(a) 485
(b) 550
(c) 520
(d) 510
(e) 490

DOWNLOAD IBPS PO Question Papers PDF

DOWNLOAD IBPS CLERK Question Papers PDF

DOWNLOAD MORE BANK EXAMS E-BOOKS

Printed Study Material for IBPS PO Exam

IBPS PO (Probationary Officer) Exam Study Materials

IBPS / SBI Special TX: 
General: 

(Papers) IBPS PO Exam Paper - 2015 "Held on: 4-10-2015" ::ENGLISH LANGUAGE::

IBPS logo


(Papers) IBPS PO Exam Paper - 2015 "Held on: 4-10-2015"

::ENGLISH LANGUAGE::


DIRECTIONS (Qs. 1-5) : Rearrange the following six sentences (A), (B), (C), (D), (E) and (F) in the proper sequence to form a meaningful paragraph; then answer the questions given below them.

(A) Two of the best-performing major economies in 2014 were China and Brazil, with growth estimated at 7.5% and 10.5% resepectively.
(B) Despite that limp growth, major US stock market indexes are up between 11% and 20% for the year.
(C) Even knowing where economies are headed sometimes it is of no help to an investor.
(D) It is hard to anticipate the direction of financial markets.
(E) But as of December, stock markets of both nations were in the red for the year.
(F) By contrast, the US economy is likely to have expanded at only about 2.6% for the year.

1. Which of the following would be the SECOND sentence?

(a) A
(b) C
(c) D
(d) E
(e) F

2. Which of the following would be the FOURTH sentence?

(a) A
(b) B
(c) C
(d) E
(e) F

3. Which of the following would be the FIFTH sentence ?

(a) A
(b) B
(c) C
(d) D
(e) F

4. Which of the following would be the FIRST sentence ?

(a) B
(b) C
(c) D
(d) E
(e) F

5. Which of the following would be the SIXTH (LAST) sentence ?

(a) A
(b) B
(c) C
(d) D
(e) E

DIRECTIONS (Qs. 6-10) : Read each sentence to find out whether there is any grammatical error in it or a wrong word has been used. The error, if any, will be in one part of the sentence which has been printed in bold and has been numbered (a), (b), (c) or (d). The number of that part is the answer. If there is no error, the answer is (e) i.e. ‘No error’. (Ignore the errors of punctuation, if any.)

6. The convergence of

(a) Indian accounting standards with International Financial Reporting Standards (IFRS) beginning
(b) in April is expecting to 
(c) see power companies struggling with
(d) significant first-timeadoption impact.
(e) No error

7. Researchers at

(a) the Indian Institute of Science (IISc), Bangalore, are mapping
(b) India’s solar hot spots-where round-the-year
(c) sunlight makes it viable of 
(d) companies to set up solar power plants. 
(e) No error

8. Though their qualifications span a diverse

(a) range, there is an equal
(b) number of graduates and those who have just completed School, each set
(c) making up
(d) close to 30% of these households.
(e) No error

9. As if

(a) the most dangerous moment for any dictatorship is when
(b) it starts to
(c) reform, North Korea looks ready to turn that truism on its head.
(d) North Korea looks ready to turn that truism on its head.
(e) No error

10. It so happens

(a) that this happy campy ritual is their way of life
(b) and one into which
(c) they don’t particularly welcome
(d) voyeuristic intrusions. 
(e) No error

DIRECTIONS (Qs. 11-20) : Read the following passage carefully and answer the questions given below it. Certain words/ phrases are printed in bold to help you to locate them while answering some of the questions.

The outside world has pat answers concerning extremely impoverished countries, especially those in Africa. Everything comes back, again and again, to corruption and mis-rule. Western officials argue that Africa simply needs to behave itself better, to allow market forces to operate without interference by corrupt rulers. Yet the critics of African governance have it wrong. Politics simply can't explain Africa's prolonged economic crisis. The claim that Africa's corruption is the basic source of the problem does not withstand serious scrutiny. During the past decade I witnessed how relatively well-governed countries in Africa, such as Ghana, Malawi, Mali and Senegal, failed to prosper, whereas societies in Asia perceived to have extensive corruption, such as Bangladesh, Indonesia and Pakistan, enjoyed rapid economic growth. What is the explanation? Every situation of extreme poverty around the world contains some of its own unique causes, which need to be diagnosed as a doctor would a patient. For example, Africa is burdened with malaria like no other part of the world, simply because it is unlucky in providing the perfect conditions for that disease; high temperatures, plenty of breeding sites and particular species of malaria-transmitting mosquitoes that prefer to bite humans rather than cattle. Another myth is that the developed world already gives plenty of aid to the world's poor. Former U.S. Secretary of the Treasury, Paul O'Neil expressed a common frustration when he remarked about aid for Africa : "We've spent trillions of dollars on these problems and we have damn near nothing to show for it". O'Neil was no foe of foreign aid. Indeed, he wanted to fix the system so that more U.S. aid could be justified. But he was wrong to believe that vast flows of aid to Africa had been squandered. President Bush said in a press conference in April 2004 that as "the greatest power on the face of the earth, we have an obligation to help the spread of freedom. We have an obligation to feed the hungry". Yet how does the U.S. fulfill its obligation? U.S. aid to farmers in poor countries to help them grow more food runs at around $200 million per year, far less than $1 per person per year for the hundreds of millions of people living in subsistence farm households. From the world as a whole, the amount of aid per African per year is really very small, just $30 per sub-Saharan African in 2002. Of that modest amount, almost $5 was actually for consultants from the donor countries, more than $3 was for emergency aid, about $4 went for servicing Africa's debts and $5 was for debt-relief operations. The rest, about $12, went to Africa. Since the "money down the drain" argument is heard most frequently in the U.S., it's worth looking at the same calculations for U.S. aid alone. In 2002, the U.S. gave $3 per sub-Saharan African. Taking out the parts for U.S. consultants and technical cooperation, food and other emergency aid, administrative costs and debt relief, the aid per African came to grand total of 6 cents. The U.S. has promised repeatedly over the decades, as a signatory to global agreements like the Monterrey Consensus of 2002, to give a much larger proportion of its annual output, specifically upto 0.7% of GNP, to official development assistance. The U.S. failure to follow through has no political fallout domestically, of course, because not one in a million U.S. citizens even knows of statements like the Monterrey Consensus. But no one should underestimate the salience that it has around the world. Spin as American might about their nation's generosity, the poor countries are fully aware of what the U.S. is not doing.

11. The passage seems to emphasize that the outside world has

(a) correct understanding about the reasonable aid provided by the USA to the poor countries
(b) definite information about what is happening in under developed countries
(c) stopped extending any financial aid to under developed countries
(d) misconceptions about the aid given to the poor nations by developed countries
(e) None of these

12. According to the Westerners the solution to eradicate poverty of African nations lies in

(a) corruption
(b) improving their own national behaviour
(c) mis-rule
(d) prolonged economic crisis
(e) None of these

13. The author has given the example of Bangladesh, Indonesia and Pakistan in support of his argument that

(a) corruption is the major culprit in the way of prosperity
(b) mis-governance hampers the prosperity of nations
(c) despite rampant corruption, nations may prosper
(d) developed nations arrogantly neglect under developed countries.
(e) None of these

14. The author has mentioned Ghana as a country with

(a) reasonably good-governance
(b) corrupt leadership
(c) plenty of natural resources
(d) rapid economic growth
(e) None of these

15. The cases of malaria in Africa are mainly due to

A. high temperature.
B. climatic conditions conducive for breeding.
C. malaria carries liking for human blood in preference to that of cattle.

(a) None of these
(b) Only B and C
(c) Only A and C
(d) Only A and B
(e) All the three

DOWNLOAD IBPS PO Question Papers PDF

DOWNLOAD IBPS CLERK Question Papers PDF

DOWNLOAD MORE BANK EXAMS E-BOOKS

Printed Study Material for IBPS PO Exam

IBPS PO (Probationary Officer) Exam Study Materials

IBPS / SBI Special TX: 
Subject: 
General: 

(Papers) IBPS PO Exam Paper - 2016 "Quantitative Aptitude Held on: 16-10-2016"

IBPS logo


(Papers) IBPS PO Exam Paper - 2016 "Held on: 16-10-2016"

:: Quantitative Aptitude ::


1. If the male population above poverty line for State R is then what is the total population of

(1) 4.5 Trillion
(2) 4.85 million
(3) 5.33 million
(4) 6.25 million
(5 ) 6 million

2. What will be the number of females above poverty line in the State S if it is known that the population of State S is 7 million ?

(1) 3 million
(2) 2.13 million
(3) 1.33 million
(4) 5.7 million
(5) 4 million

3. What is the male population above poverty line for State P if the female population below poverty line for State P is 2.1 million ?

(1) 2.1 million
(2) 2.7 million
(3) 3.3 million
(4) 2.3 million
(5) 3 million

4. If the population of males below poverty line for State Q is 2.4 million and that for State T is 6 million, then what is the respective ratio of the total population of states Q and T ?

(1) 1 : 3
(2) 2 : 5
(3) 3 : 7
(4) 4 : 9
(5) 3 : 9

5. Find the probability that a number from 1 to 300 is divisible by 3 or 7 ?

(1) 37/75
(2) 32/75
(3) 36/75
(4) 28/75
(5) 26/75 

6. 14 men can do a work in 18 days, 15 women can do a work in 24 days. If 14 men work for first three days and 10 women work after that for three days, find the part of work left after that ?

(1) 3/4
(2) 1/4
(3) 1/2
(4) 1/6
(5) 1/5

7. Perimeter of a rectangle is x and circumference of a circle is 8 more than the perimeter of the rectangle. Ratio of radius of circle and length of the rectangle is 1:2 and ratio of length and breadth of rectangle is 7:3. Find the length of the rectangle?

(1) 14
(2) 21
(3) 28
(4) 35
(5) 7

8. A invests on some scheme at 5% and B at 3% for two years. If the total sum invested by A and B is 4000 and the simple interest received by both is same then find the amount invested by A.

(1) Rs. 1,300/-
(2) Rs. 1,500/-
(3) Rs. 2,500/-
(4) Rs. 2,700/-
(5) Rs. 2,100/-

9. Two trains crosses each other in 14 sec when they are moving in opposite direction, and when they are moving in same direction they crosses each other in 3 min 2 sec. Find the speed of the faster train by what percent more than the speed of the slower train ?

(1) 16.67%
(2) 17.33%
(3) 16.33%
(4) 17.67%
(5) 18.33%

10. 11 20 38 74 ?

(1) 146
(2) 154
(3) 128
(4) 132
(5) 136

11. 15 21 38 65 101 ?

(1) 124
(2) 145
(3) 136
(4) 158
(5) 162

12. 24 28 19 35 10 ?

(1) 26
(2) 36
(3) 16
|(4) 46
(5) 15

13. 7 16 45 184 915 ?

(1) 2092
(2) 5496
(3) 1048
(4) 4038
(5) 3268

14. 12 19 35 59 90 ?

(1) 134
(2) 127
(3) 132 (4) 98
(5) 114

DOWNLOAD IBPS PO Question Papers PDF

Printed Study Material for IBPS PO Exam

IBPS PO (Probationary Officer) Exam Study Materials

IBPS / SBI Special TX: 
IBPS HINDI - आई. बी. पी. एस.: 
Subject: 
General: 

(Papers) IBPS PO Exam Paper - 2016 "English Held on: 16-10-2016"

IBPS logo

(Papers) IBPS PO Exam Paper - 2016 "Held on: 16-10-2016"

:: English Language ::

Directions (1-7) : Read the passage carefully and answer the questions given below it.

Governments looking for easy popularity have frequently been tempted into announcing give-a-ways of all sorts; free electricity, virtually free water, subsidized food, cloth at half price, and so on. The subsidy culture has gone to extremes. The richest farmers in the country get subsidized fertilizers. University education, typically accessed by the wealthier sections, is charged at a fraction of cost. Postal services are subsidized, and so are railway services. Bus fares cannot be raised to economical levels because there will be violent protest, so bus travel is subsidized too. In the past, price control on a variety of items, from steel to cement, meant that industrial consumer of these items got them at less than actual cost, while the losses of the public sector companies that produced them were borne by the taxpayer! A study done a few years ago, came to the conclusion that subsidies in the Indian economy total as much as 14.5 per cent of gross domestic product. At today’s level, that would work out to about 1,50,000 crore. And who pay the bill? The theory-and the Political fiction on the basis of I which it is sold to unsuspecting voters-is that subsidies go the poor. and are paid for by the rich. The fact is that most subsidies go the ‘rich’ (defined in the Indian context as those who are above the poverty line), and much of the tab goes indirectly to the poor. Because the hefty subsidy bill results in fiscal deficits, which in turn push up rates of inflation-which, as everyone knows, hits the poor the hardest of all. That is why taxmen call inflation the most regressive form of taxation.

The entire subsidy system is built on the thesis that people cannot help themselves, therefore governments must do so. That people cannot afford to pay for variety of goods and services, and therefore the government must step in. This thesis has been applied not just in the poor countries but in the rich ones as well; hence the birth of the welfare state in the west, and an almost Utopian social security system; free medical care, food aid, old age security, et.al. But with the passage of time, most of the wealthy nations have discovered that their economies cannot sustain this social safety net, which in fact reduces the desire among people to pay their own way, and takes away some of the incentive to work, in short, the bill was unaffordable, and their societies were simply not willing to pay. To the regret of many, but because of the laws of economies are harsh, most Western societies have been busy pruning the welfare bill.

In India, the lessons of this experience over several decades, and in many countries-do not seem to have been learnt. Or they are simply ignored in the pursuit of immediate votes. People who are promised cheap food or clothing do not in most cases look beyond the gift horses-to the question of who picks up the tab. The uproar over higher petrol, diesel and cooking gas prices ignored this basic question; if the user of cooking gas does not want to pay for its cost, who should pay? Diesel in the country is subsidised, and if the user of cooking gas does not want to pay for its full cost, who does he or she think should pay the balance of the cost? It is a simple question, nevertheless if remains unasked.

The Deva Gowda government has shown some courage in biting the bullet when it comes to the price of petroleum products. But it has been bitten by much bigger subsidy bug. It wants to offer food at half its
cost to everyone below the poverty line, supposedly estimated at some 380 million people. What will be the cost? And of course, who will pick up the tab? The Andhra Pradesh Government has been bankrupted by selling rice as 2 per kg. Should the Central Government be bankrupted too, before facing up to the question of what is affordable and what is not? Already, India is perennially short of power because the subsidy on electricity has bankrupted most electricity boards, and made private investment wary unless it gets all manner of state guarantees. Delhi’s subsidised bus fares have bankrupted the Delhi Transport Corporation, whose buses have slowly disappeared from the capital’s streets. It is easy to be soft and sentimental, by looking at programmes that will be popular. After all, who does not like a free lunch? But the evidence is surely mounting that the lunch isn’t free at all. Somebody is paying the bill. And if you want to know who, take at the country’s poor economic performance over the years.

1. Which of the following should not be subsidised over the years ?

(1) University education
(2) Postal services
(3) Steel
(4) Other than those given as options
(5) All of the above options

2. The statement that ‘subsidies are paid by the rich and go to the poor’ is

(1) fiction
(2) fact
(3) fact, according to the author
(4) fiction, according to the author
(5) Other than those given as options

3. Why do you think that the author calls the Western social security system Utopian ?

(1) The countries belief in the efficacy of the system was bound to turn out to be false.
(2) The system followed by these countries is the best available in the present context.
(3) Everything under this system was supposed to be free but people were charging money for them.
(4) The theory of system followed by these countries was devised by Dr. Utopia.
(5) All the options are responsible.

4. It can be inferred from the passage that the author :

(1) believes that people can help themselves and do not need the government.
(2) believes that the theory of helping with subsidy is very destructive.
(3) believes in democracy and free speech.
(4) is not a successful politician.
(5) believes that subsidies are the best way to help poor.

5. Which of the following is not a victim of extreme subsidies ?

(1) The poor
(2) The Delhi-Transport Corporation
(3) The Andhra Pradesh Goverment
(4) Other than those given as options
(5) The rich

6. Which of the following is not true in the context of the passage ?

(1) Where subsidies are concerned, the poor ultimately pay the tab.
(2) Inflation is caused by too much subsidies.
(3) Experts call subsidies the most regressive form of taxation.
(4) Fiscal deficits are caused due to heavy subsidy bills.
(5) None of the following is true in the context of the passage.

7. A suitable title to the passage would be :

(1) There’s no such thing as a free lunch
(2) The Economic Overview
(3) Deva Gowda’s Government and its Follies
(4) It takes Two to Tango
(5) The Rich and The Poor: Extreme Partiality

Directions (8-12) : Rearrange the following six sentences (A), (B), (C), (D), (E) and (F) in the proper sequence to form a meaningful paragraph and then answer the questions given below.

A. It is the only country in the world that is carbon negative, which means it produces more oxygen than it consumes.
B. Bhutan, sandwiched between the two most populous nations on Earth, suffers for their sins.
C. So far, so good. But then, two things happened.
D. Carbon sinks, 70% forest cover, powered almost entirely by mountain streams—Bhutan is a poster child for green living.
E. Glaciers are beginning to melt, flash floods and heavy rains—and even droughts— are common, and temperatures are climbing.
F. One, India and China got richer.

8. Which of the following should be the FIRST sentence of the given paragraph ?

(1) E
(2) D
(3) C
(4) B
(5) A

9. Which of the following should be the THIRD sentence of the given paragraph ?

(1) A
(2) B
(3) C
(4) D
(5) E

10. Which of the following should be the LAST sentence of the given paragraph ?

(1) A
(2) D
(3) C
(4) B
(5) E

11. Which of the following should be the FOURTH sentence of the given paragraph?

(1) F
(2) C
(3) B
(4) E
(5) D

12. Which of the following should be the SECOND sentence of the given paragraph?

(1) B
(2) D
(3) A
(4) C
(5) E

DOWNLOAD IBPS PO Question Papers PDF

Printed Study Material for IBPS PO Exam

IBPS PO (Probationary Officer) Exam Study Materials

IBPS / SBI Special TX: 
IBPS HINDI - आई. बी. पी. एस.: 
Subject: 
General: 

(Papers) IBPS PO Exam Paper - 2016 "Reasoning Held on: 16-10-2016"

IBPS logo


(Papers) IBPS PO Exam Paper - 2016 "Held on: 16-10-2016"

:: Reasoning Ability ::


Directions (1-5) : In these questions, relationship between different elements is shown in the statements. The statements are followed by conclusions. Study the conclusions based on the given statements and select the appropriate answer.

Give answer :

(1) If only conclusion II is true
(2) If only conclusion If is true
(3) If both conclusions I and II are true
(4) I f either conclusion I or II is true
(5) If neither conclusion I nor II is true

1. Statements :

S d” L d” I = P e” E > R ; L > Q

Conclusions :

I. P > S
II. I > R

Directions (6-10) Study the following information carefully and answer the questions given below.

Eight persons — H, I, J, K, L, M, N and O — are standing in a straight line at equidistant. Some of them are facing north while others are facing south.
M is standing third to the right to H. M is standing at one of the extreme ends. L is standing third to the left of H. The immediate neighbours of J. face north. N is not an immediate neighbour of H.The persons standing at the extreme ends face the same direction (both are facing either north or south). The immediate neighbours of H face, just opposite direction as that of M. The immediate neighbours of 0 face opposite direction with respect to each other.K is one of the immediate neighbours of L and is facing north. I is standing between J and M. Not more than four persons are facing north.L is immediate right of K.

6. Who among the following is third to the left of N?

(1) K
(2) J
(3) H
(4) I
(5) O

7. The immediate neighbours of L are :

(1) M and N
(2) N and O
(3) K and N
(4) N and H
(5) J and H

8. How many persons are standing exactly between I and 0 ?

(1) Three
(2) Four
(3) One
(4) Two
(5) None

9. Four of the following five are alike in a certain way based on the above arrangement and hence form a group. Which of the following does not belong to that group ?

(1) N
(2) L
(3) O
(4) J
(5) K

10. Who among the following is exactly between L and J ?

(1) N
(2) O
(3) H
(4) I
(5) None

Directions (11-15) : In each question below are given two/three statements followed by two conclusions numbered I and II. You have to take the given statements to be true even if they seem to be at variance with commonly known facts. Read all the conclusions and then decide which of the given conclusions logically follows from the given statements, disregarding commonly known facts. Give answer :

(1) If only conclusion I follows.
(2) If only conclusion II follows.
(3) If either conclusion I or II follows.
(4) If neither conclusion I nor II follows.
(5) If both conclusions I and II follow.

11. Statements :

All circles are a triangle.
Some triangles is rectangles.
All rectangles are squares.

Conclusions :

I. All rectangles being triangles is a possibility.
II. All circles being squares is a possibility.

12. Statements :

Some chairs are tables.
Some beds are tables.
No furniture is bed.

Conclusions :

I. All chairs being furniture is a possibility.
II. Some tables are not bed is a possibility.

13. Statements :

All circles are a triangle.
Some triangles are rectangles.
All rectangles are squares.

Conclusions :

I. Some triangles are not rectangles.
II. No square is a circle.

14. Statements :

All art are theatre.
Some art are dramas

Conclusions :

I. All dramas being theatre is a possibility.
II. Some dramas are theatre.

15. Statements :

Some chairs are tables.
Some beds are tables.
No furniture is bed.

Conclusions :

I. Some tables are not furniture.
II. All tables being furniture is a possibility.

DOWNLOAD IBPS PO Question Papers PDF

DOWNLOAD IBPS CLERK Question Papers PDF

Printed Study Material for IBPS PO Exam

IBPS PO (Probationary Officer) Exam Study Materials

IBPS / SBI Special TX: 

(Papers) IBPS PO Exam Paper - 2016 "Held on: 20-11-2016" ::QUANTITATIVE APTITUDE::

IBPS logo


(Papers) IBPS PO Exam Paper - 2016 "Held on: 20-11-2016"

::QUANTITATIVE APTITUDE::


1. Ankita borrows ` 7000 at simple Interest from a lender. At the end of 3 years, she again borrows ` 3000 and settled that amount after paying ` 4615 as interest after 8 years
from the time she made the first borrowing. What is the rate of interest?

(a) 5.5%
(b) 9.5%
(c) 7.5%
(d) 6.5%
(e) None of the Above

2. The difference between compound interest compounded every 6 months and simple interest after 2 years is 248.10. The rate of interest is 10 percent. Find the sum

(a) 12000
(b) 14000
(c) 16000
(d) 18000
(e) None of these

3. Deepak found that he had made a loss of 10% while selling his smartphone. He also found that had he sold it for ` 50 more, he would have made a profit of 5%. The initial loss was what percentage of the profit earned, had he sold the smartphone for a 5% profit ?

(a) 100%
(b) 200%
(c) 75%
(d) 85%
(e) None of the Above

4. An inspector is 228 meter behind the thief. The inspector runs 42 meters and the thief runs 30 meters in a minute. In what time will the inspector catch the thief?

(a) 19 minutes
(b) 20 minutes
(c) 18 minutes
(d) 21 minutes
(e) None of these

5. Ashok can row upstream at 8 kmph and downstream at 12 kmph.What is the speed of the stream ?

(a) 6 km/hr
(b) 3 km/h
(c) 2 km/hr
(d) 4 km/hr
(e) 4.5 km/hr

6. James' present age is 2/7th of his father's present age. James' brother is three year older to James. The respective ratio between present ages of James' father and James' brother is 14:5. What is the present age of James?

(a) 12 years
(b) 23 years
(c) 19 years
(d) 27 years
(e) 13 years

7. The average value of property of Agil, Mugilan and Anitha is `130cr. The Property of Agil is 20cr greater than the property value of Mugilan and Anitha property value is
50cr greater than the Agil property value. The value of property of Anitha is

(a) 120 cr
(b) 170 cr
(c) 100 cr
(d) 150 cr
(e) None of these

8. A and B can do a piece of work in 24 and 30 days respectively. Both started the work and worked for 6 days. Then B leaves the work and C joins and the remaining work
is completed by A and C together in 11 days. Find the days in which C alone can do the work

(a) 80
(b) 100
(c) 120
(d) 130
(e) None of these

9. Three pipe P, Q and R can fill a tank in 12 minutes, 18 minutes and 24 minutes respectively. The pipe R is closed 12 minutes before the tank is filled. In what time the tank is full?

10. A company reduces his employee in the ratio 14 : 12 and increases their wages in the ratio 16:18. Determine whether the bill of wages increases or not and in what ratio.

(a) Decreases, 28: 27
(b) Increases, 27:28
(c) Decreases, 29:28
(d) Increases, 28:29
(e) None of these

11. A slice from a circular pizza of diameter 14 inches is cut in a such a way that each slice of pizza has a central angle of 45°. What is the area of each slice of Pizza(in square inches)?

(a) 16.25
(b) 19.25
(c) 18.25
(d) 17.25
(e) None of the Above

12. 8 litres are drawn from a flask containing milk and then filled with water. The operation is performed 3 more times. The ratio of the quantity of milk left and total solution is 81/625. How much milk the flask initially holds?

(a) 10 ltr
(b) 20 ltr
(c) 30 ltr
(d) 40 ltr
(e) None of these

13. Two persons A and B invested in a business with 115000 and 75000 rupees respectively. They agree that 40% of the profit should be divided equally among them and rest is divided between them according to their investment. If A got 500 rupee more than B, then the total profit is.

(a) 3599.34
(b) 699.34
(c) 3958.34
(d) 999.34
(e) None of these

14. The ratio of the monthly salaries of A and B is in the ratio 5 : 16 and that of B and C is in the ratio 17 : 18. Find the monthly income of C if the total of their monthly salary is ` 1,87,450.

(a) ` 66,240
(b) ` 72,100
(c) ` 62,200
(d) ` 65,800
(e) ` 60,300

15. Pipes A and B can fill a cistern in 15 hours together. But if these pipes operate separately A takes 40 hours less than B to fill the tank. In how many hours the pipe A will fill the cistern working alone?

(a) 60
(b) 20
(c) 40
(d) 15
(e) 25

DIRECTIONS (Qs. 16-20) : In these questions, a number series is given. Find out the missing number.

16. 20 39 74 ? 262 499

(a) 146
(b) 169
(c) 166
(d) 139
(e) 129

17. 14 8 6 6 7 ?

(a) 6.5
(b) 7.5
(c) 8.5
(d) 9.5
(e) None of these

18. 6 13 20 65 256 ?

(a) 1283
(b) 1756
(c) 2786
(d) 2686
(e) None of these

19. 14 6 4 4 8 ?

(a) 26
(b) 32
(c) 46
(d) 27
(e) None of these

20. 6 16 45 184 917 ?

(a) 5506
(b) 4756
(c) 5786
(d) 5686
(e) None of these

DIRECTIONS (Qs. 21-25) : In the following questions two equations numbered I and II are given. You have to solve both the equations and give answer.

(a) x > y
(b) x < y
(c) x y
(d) x ≥ y
(e) x = y or relation cannot be established

22. 5x + 2y = 31
3x + 7y = 36

(a) x > y
(b) x < y
(c) x y
(d) x≥ y
(e) x = y or relation cannot be established

23. 7x + 6y + 4z = 122
4x + 5y + 3z = 88
9x + 2y + z = 78

(a) x < y = z
(b) x ≥ y < z
(c) x < y > z
(d) x = y > z
(e) x = y = z or relation cannot be established

24. 4x + 2y =8.5, 2x + 4y = 9.5

(a) x > y
(b) x < y
(c) x y
(d) x ≥ y
(e) x = y or relation cannot be established

25. I. 56×2 + 37x + 6 = 0
II. 66y2 – 13y – 4 = 0

(a) if x y
(b) if x > y
(c) if x < y
(d) if x ≥ y
(e) if the relation between x and y cannot be established.

DOWNLOAD IBPS PO Question Papers PDF

DOWNLOAD IBPS CLERK Question Papers PDF

Printed Study Material for IBPS PO Exam

IBPS PO (Probationary Officer) Exam 2018 Study Materials

IBPS / SBI Special TX: 
General: 

(Papers) IBPS PO Exam Paper - 2016 "Held on: 20-11-2016" ::REASONING ABILITY::

IBPS logo

(Papers) IBPS PO Exam Paper - 2016 "Held on: 20-11-2016"

::REASONING ABILITY::

DIRECTIONS (Qs. 1-5) : Read the following infomation carefully and answer the questions that follow.

1. Statements: Some trees are plants. All trees are houses. No plant is pot. All tools are pots.

Conclusions:

I. Some plants are tools is a possibility.
II. All trees are tools is a possibility.
III. No plant is house.
IV. All plants are houses is a possibility.

(a) Only I and III follow.
(b) Either III or IV follows
(c) Only IV follows
(d) None follows.

2. Statements: All trees are plants. All houses are trees. Some pots are plants. All tools are plants.

Conclusions:

I. Some plants are not houses
II. Some tools being trees is a possibility
III. All pots are tools is a possibility.
IV. All houses are tools is a possibility.

(a) Only III and IV follow
(b) Only II, III, and IV follow
(c) Only I and IV follow.
(d) All follow.

3. Statements: Some roofs are figures. All figures are lions. All lions are goats. No tool is lion.

Conclusions:

I. Some tools are figures is a possibility.
II. All tools being goats is a possibility.
III. No roof is tool.
IV. Some goats are figures

(a) Only II and IV follow
(b) Only I and II follow
(c) Only either I or II and IV follow
(d) None follows.

4. Statements : No photo is frame. All frames are sites. Some buckets are photos. Some sites are buckets.

Conclusions:

I. No photo is site.
II. All buckets being frames is a possibility
III. Some sites are photos
IV. Some buckets are frames.

(a) All follows
(b) Only II and III follow
(c) Only II, III and IV follow.
(d) Only either I or III follows

5. Statements: All seats are covers. No cover is bun. Some buns are roofs. Some sites are covers.

Conclusions:

I. All sites being buns is a possibility
II. All sites being roof is a possibility
III. Some sites are not buns.
IV. No seat is bun

(a) Only II, III and either I or IV follow.
(b) Only III follows
(c) All follows.
(d) Only II, III and IV follow

DIRECTIONS (Qs. 6-10) : Study the information below and answer the following question:-

In a certain code language,
'Arrive today eagles later' is written as 21×R,6$A, 14$O, 25×A
'Begin work faster table' is written as 14$A, 17%O, 26×A, 22$E
'Length error arrow burn' is written as 6×E, 25$R, 22%U, 21$R
'Trial better than wisdom' is written as 14$R, 14%H, 22×E, 17×I

6. The code for the word ' Table'

(a) 26×A
(b) 17%O
(c) 14$A
(d) 22$ E
(e) None of these

7. The code word 6$A for the word

(a) Later
(b) Arrive
(c) Earlier
(d) Today
(e) Either 1 or 3

8. Find the code word for the word 'Burn' ?

(a) 25$R
(b) 22%U
(c) 21$R
(d) 6×E
(e) Can't be determined

9. According to the given code, what is the code for 'M' ?

(a) 12
(b) 8
(c) 10
(d) 7
(e) Can't be determined

10. By using the given code word, find the code word for 'Better Luck Next Arrive' ?

(a) 22$E, 21$R, 6×U, 8% E
(b) 8×E, 21×R, 22%E, 6%U
(c) 22×E, 6%U, 8%E, 21×R
(d) 21%R, 22$E, 6×U, 8%E
(e) 6$U, 22×E, 8×E, 21%R

11. Two Person P and Q are separated by a distance of 20 meter in west-east direction respectively. Now P and Q start walking in north and south direction respectively and walked for 5 meter. Now P and Q took a right turn and walked 10m each. Now P and Q took left turn and after walking 5 meter both of them stopped. Find the distance between them

(a) 15
(b) 25
(c) 30
(d) 35
(e) None of these

DIRECTIONS (Qs. 12-16) : Study the following information carefully to answer the given questions.

Eight friends A, B, C, D, E, F, G and H are sitting around a circular table but not necessarily in the same order. Some of them are facing outward. They are working in four different companies Apple, IBM, Google and Intel. Two persons are working at each company.
G sits on the immediate right of B, who works at the Google. C sits third to the left of H, who works at the Apple and both are facing the same direction. C and B are not facing the same direction but C is an immediate neighbor of E, who is fourth to the left of G. E and G both are facing opposite directions but both work at the same company. Those who work at the Google sit adjacent to each other but face opposite direction. Those who work at the IBM sit opposite each other. The immediate neighbours of E are
not facing outward. A person who works at the Apple is an immediate neighbor of the persons who work at the Intel. D and F are immediate neighbours of H. D is not facing the centre and works at the Intel. The one who is on the immediate left of F is not facing the centre. F sits second to the right of C.

12. Who among the following works at the Apple ?

(a) D and F
(b) H and F
(c) G and C
(d) C and H
(e) None of these

13. Who among the following sits on the immediate right of the person who works at the IBM?

(a) B
(b) D
(c) A
(d) F
(e) None of these

14. How many persons are facing outward?

(a) Two
(b) Three
(c) Four
(d) Can't be determined
(e) None of these

15. A works at which of the following?

(a) Either Google or Apple
(b) Either Intel or IBM
(c) Google
(d) Intel
(e) Can't be determined

16. If D and F interchange their places then who among the following is on the immediate left of G?

(a) B
(b) D
(c) H
(d) F
(e) None of these

DIRECTIONS (Qs. 17-21) : In the following Questions, $, ×, %, @ and © are used with the following meaning as illustrated below.

A × B means A is not greater than B
A @ B means A is neither greater than nor equal to B
A © B means A is not smaller than B
A % B means A is neither smaller than nor greater than B
A $ B means A is neither smaller than nor equal to B

17. Statement : X × W, W @ Z , Z % Y

Conclusion:

I.Y $ W
II.Z $ W
III.X © Y

(a) Only I is true
(b) Ony II is true
(c) Both I and II are true
(d) Both I and III are true
(e). None is true

18. Statement : D $ E, E © C, C @ A

Conclusion :

I. C × D
II. A $ E
III. A $ D

(a) Only I is true
(b) Ony II is true
(c) Both I and II are true
(d) Both I and III are true
(e) None is true

19. Statement : M @ O, O % P, P © N

Conclusion:

I. P © M
II. N @ O
III. N % O

(a) Only I is true
(b) Ony II is true
(c) Both II and III are true
(d) Either II or III is true
(e) None is true

20. Statement : A © C, C $ B, B % E

Conclusion :

I. E @ C
II. B @ A
III. A © E

(a) Only I is true
(b) Ony II is true
(c) Both I and II are true
(d) Both I and III are true
(e) None is true

21. Statement : P © S, S $ R, R × Q

Conclusion :

I. Q $ S
II. R @ P
III. R % P

(a) Only I is true
(b) Ony II is true
(c) Both I and II are true
(d) Either II or III is true
(e) All I, II and III are true

DIRECTION (Qs. 22-25) : Read the following information carefully & answer the given questions that follows.

In a family there are 6 members P, Q, R, X, Y and Z. R is the sister of Z. X is the father of P and grandfather of Z. Q is the brother of Y's husband. There are three brothers, one mother and two fathers.

22. Who is the mother of R?

(a) Q
(b) Z
(c) Y
(d) X
(e) None of these

23. Who is Y's husband ?

(a) Q
(b) P
(c) X
(d) Can't be determined
(e) None of these

24. How many females are there in the family?

(a) One
(b) two
(c) three
(d) four
(e) None of these

25. Which of the following is the group of brothers ?

(a) PQZ
(b) PRZ
(c) XZP
(d) QRZ
(e) None of these

DOWNLOAD IBPS PO Question Papers PDF

DOWNLOAD IBPS CLERK Question Papers PDF

DOWNLOAD MORE BANK EXAMS E-BOOKS

Printed Study Material for IBPS PO Exam

IBPS PO (Probationary Officer) Exam 2018 Study Materials

IBPS / SBI Special TX: 
General: 

(Papers) IBPS PO Exam Paper - 2016 "Held on: 20-11-2016" ::ENGLISH LANGUAGE::

IBPS logo


(Papers) IBPS PO Exam Paper - 2016 "Held on: 20-11-2016"


::ENGLISH LANGUAGE::

DIRECTIONS (Qs. 1-5) : Read the following passage carefully and answer the questions given below it.

The finance ministry on Monday said the Union budget would be growth-oriented, implicitly signaling that it will address the investment crisis in the Indian economy. “Given the fiscal constraints and other parameters under which the government has to function, the effort of the government is to present a budget which is growthoriented, that maintains the momentum of growth and tries to develop on it,” economic affairs secretary Shaktikanta Das said in an interview with DD News uploaded on YouTube on Monday.
According to Das, the budget will also detail new measures to support ongoing programmes such as Start-up India, Standup India, Make In India, Digital India and the Skill mission – all of which have a strong focus on creating jobs. Finance minister ArunJaitley will be presenting his third budget on 29 February at a time when private investment has dried up and the exchequer has had to incur higher expenditure due to implementation of the One Rank One Pension scheme for the armed forces and the commendations of the Seventh Pay Commission. That may cramp the government’s ability to accelerate public investment to revive economic growth while sticking within the confines of
its fiscal deficit targets. Some parts of the government believe that the emphasis should be on growth and not fiscal consolidation. Other parts, and the Reserve Bank of India, believe the finance minister should adhere to his fiscal commitments made in the last budget. Without revealing whether the government will digress from the path of fiscal consolidation, Das said the government’s priority is to take a balanced view on “the expenditure requirement to keep our growth momentum and to what extent we can borrow’. Care Ratings chief economist MadanSabnavis said the government has to increase its allocation for public investment on infrastructure to stimulate growth. “I expect government to spend ` 10,000-20,000 crore additional amount on infrastructure. Given nominal GDP (gross domestic product) is not expected to expand significantly, the leeway for the government to spend more may not be there while keeping fiscal deficit within 3.7-3.9% of GDP. So I don’t expect a big-bang push for infrastructure
spending given the fiscal constraint,” he said. The finance ministry revealed more contours of its budget when minister of state for finance JayantSinha, also in an interview
to DD News, said the four pillars of the budget will be poverty eradication, farmers’ prosperity, job creation and a better quality of life for all Indian citizens. ‘This budget will be a forward looking budget that will ensure that India will continue to be a haven of stability and growth in a very turbulent and choppy global economic environment,” he added. The government has been contemplating tax incentives to companies in the manufacturing sector, including tax deductions on emoluments paid to new employees, to encourage firms to step up hiring and create jobs under its Make in India initiative. The government published suggestions that it has received internally from various government departments and other stakeholders on the mygov.in website, seeking further ideas and comments from the public. Suggestions being considered by the
government include financial incentives, tax incentives under the Income Tax Act, 1961, and subsidies for equipping employees with job skills, and upgrading and improving employment exchanges. Another suggestion is to expand the scope of the tax deduction currently available to companies that add at least 10% to their workforce in a year by lowering the threshold. This incentive is available only in cases of employees who earn less than ` 6 lakh a year.

1. What is the main objective of the government to create the Union Budget?

(a) It should meet the requirements of the society.
(b) It should be under some fiscal constraints.
(c) It should be growth oriented.
(d) It should meet the requirements of a developed country.
(e) It should change the momentum of growth.

2. Where is it expected to invest by government to stimulate growth?

(a) On infrastructure
(b) On governments plans.
(c) On fiscal management
(d) On manufacturing sector
(e) On social development.

3. What does this mean that India will continue to be a “haven of stability”?

(a) That new budget will make India stable forever.
(b) The four pillars of budget will lead to make stability.
(c) India will continue towards stability even in disturbed economic environment.
(d) The budget will remain unchanged even in turbulent and choppy economic environment.
(e) None of the above

4. Why is the government providing tax incentive to companies in manufacturing sector?

(a) For better infrastructure.
(b) For tax deductions on emoluments paid to new employees,
(c) To create new job opportunities and to initiate project ‘Make in India’
(d) to create new job opportunities and to initiate project Standup India.
(e) to encourage firms to step up hiring new skilled employees

5. Which one of the following is NOT the suggestion considered by the government?

(a) To expand the scope of tax deduction to companies that add at least 10% to their workforce in a year.
(b) To upgrade and improve employment exchange.
(c) To provide incentive to employees who earn less than Rs.6 lakh a year.
(d) Subsides to train employees with job skills.
(e) Tax penalty for high income people.

DIRECTIONS (Qs. 6-10) : Read the following passage carefully and answer the questions given below it.

The alarm bells should start ringing any time now. An important component of the economy has been sinking and needs to be rescued urgently. This critical piece is ‘savings’ and, within this overall head, household savings is the one critical subcomponent that needs close watching and nurturing. While it is true that one of the primary reasons behind the current economic slowdown is the tardy rate of capital expansion - or, investment in infrastructure as well as plant and machinery - all attempts to stimulate investment activity are likely to come to naught if savings do not grow. Without any growth in the savings rate, it is futile to think of any spurt in investment and, consequently, in the overall economic growth. If we source all the investment funding from overseas, it might be plausible to contemplate investment growth without any corresponding rise in savings rate. But that is unlikely to happen. Within the overall savings universe, the subcomponent ‘household savings’ is most critical. It provides the bulk of savings in the economy, with private corporate savings and government saving contributing the balance. The worrying factor is the nearstagnation in household savings over the last eight years or so.
What’s even more disconcerting is the fact that household savings remained almost flat during the go-go years of 2004-08. This seems to be counter-factual. There are many studies that show that there is a direct relationship between overall economic growth and household savings. So, at a time when India’s GDP was growing by over 9% every year, the household savings rate stayed almost constant at close to 23% of GDP. There was, of course, an increase in absolute terms, but it remained somewhat fixed as a proportion of GDP. What is responsible for this contradictory movement? The sub-group on household savings, formed by the working group on savings for the 12th Plan set up by the Planning Commission and chaired by RBI deputy governor SubirGokarn, has this to say, “...a recent study had attributed the decline in the household saving ratio in the UK during 1995-2007 to a host of factors such as declining real interest rates, looser credit conditions, increase in asset prices and greater macroeconomic stability.
While recognising that one of the key differences in the evolving household saving scenario between the UK and India is the impact of demographics (dependency ratio), anecdotal evidence on increasing consumerism and the entrenchment of (urban) lifestyles in India, apart from the easier availability of credit and improvement in overall macroeconomic conditions, is perhaps indicative of some ‘drag’ on household saving over the last few years as well as going forward.” India has another facet: a penchant for physical assets (such as bullion or land). After the monsoon failure of 2009, and the attendant rise in price levels that has now become somewhat deeply entrenched, Indians have been stocking up on gold. Consequently, savings in financial instruments dropped while those in physical assets shot up. This is also disquieting for policy planners because savings in physical assets stay locked in and are unavailable to the economy for investment activity. There is a counter view that higher economic growth does not necessarily lead to higher savings. According to a paper published by Ramesh Jangili (Reserve Bank of India Occasional Papers, Summer 2011), while economic growth doesn’t inevitably lead to higher savings, the reciprocal causality does hold true. “It is empirically evident that the direction of causality is from saving and investment to economic growth collectively as well as individually and there is no causality from economic growth to saving and (or) investment.” Whichever camp you belong to, it is beyond doubt that savings growth is a necessary precondition for promoting economic growth. The Planning Commission estimates that an investment of $1 trillion, or over 50 lakh crore, will be required for the infrastructure sector alone. And, a large part of this critical investment will have to be made from domestic savings.

6. What is the main concern of the author behind saying that ‘the alarm bells should start ringing anytime now’?

(a) The current economic growth is slowing down due to regular failure of monsoon.
(b) Due to power shortage industrial growth could not touch the target. (c) Household savings are sinking and they require to be revamped.
(d) Due to a sharp decline in real interest rates people have lost their enthusiasm to invest in govt schemes.
(e) All the above

7. What is/are the primary reasons behind the current economic slowdown?

(A) Slow rate of capital expansion
(B) Tardy investment in infrastructure as well as plant and machinery
(C) A rapid increase in the cases of corruption, and decreased FDI

(a) Only (A)
(b) Both (A) and (B)
(c) Either (A) or (C)
(d) Both (B) and (C)
(e) All (A), (B) and (C)

8. How is household savings related to overall economic growth? Give your answer in the context of the passage?

(A) Overall economic growth is directly related to household savings.
(B) Overall economic growth is inversely proportional to household savings.
(C) There is no specific relationship between overall economic growth and household savings.

(a) Only (C)
(b) Only (B)
(c) Only (A)
(d) Either (A) or (B)
(e) Either (A) or (C)

9. What was/were the reason(s) of drop in savings in financial instruments after 2009?

(a) Rise in price level of gold
(b) Decrease in real interest rates on savings in financial instruments
(c) Investment in physical assets, particularly land
(d) Only (a) and (c)
(e) Only (b) and (c)

10. Which of the following is/are the reasons of a drag on household savings in India over the last few years?

(a) Increasing consumerism
(b) Entrenchment of urban lifestyle
(c) Easier availability of credit
(d) Improvement in overall macroeconomic conditions
(e) All the above

DIRECTIONS (Qs. 11-15) : In each of the following questions five options are given, of which one word is most nearly the same or opposite in meaning to the given word in the question. Find the correct option having either same or opposite meaning.

11. PRODIGY

(a) Pauper
(b) Despondent
(c) Demure
(d) Wanton
(e) Epitome

12. NONDESCRIPT

(a) Conducive
(b) Discern
(c) Tantamount
(d) Defined
(e) Emancipate

13. SAVANT

(a) Glutton
(b) Postulant
(c) Shrink
(d) Pluck
(e) Itinerant

14. CORPULENT

(a) Lean
(b) Gaunt
(c) Emaciated
(d) Obese
(e) Nobble

15. EMBEZZLE

(a) Misappropriate
(b) Balance
(c) Remunerate
(d) Clear
(e) Perfection

DIRECTIONS (Qs. 16-20) : Which of the pair of phrases (a), (b), (c) and (d) given below should replace the phrase given in bold in the following sentence to make the sentence grammatically meaningful and correct? If the sentence is correct as it is and no correction is required, mark (e) as the answer.

16. According to author Dishantgautam, a novel is difficult to write when compared to a play is like going for an election where one has to appeal to a thousand people at a time whereas in a book one appeals to one only person.

(a) simpler, running in
(b) faster, voting through
(c) easier, running for
(d) fool proof, voting on
(e) No correction required

17. We have in America a collection speech that is neither American, Oxford English, nor colloquial English, but a mixture of all three.

(a) motley, an enhancement
(b) hybrid, a combination
(c) nasal, a blend
(d) mangled, a medley.
(e) No correction required

18. Alice Walker’s The Temple of My Familiar, far from being a tight, focused Narrative, is instead a cheaper novel that roams freely and imaginatively over a halfmillion

(a) traditional , a chronological
(b) provocative , an insensitive
(c) forceful , a concise
(d) focused , an expansive
(e) circuitous , a discursive

19. Jayashree was habitually so docile anderratic that her friends could not understand her sudden hostile her employers.

(a) accommodating, outburst against
(b) erratic, envy of
(c) truculent, virulence toward
(d) hasty, annoyance toward
(e) apologetic, hostile

20. The village headman was unlettered, but he was no fool, he could see through the mystery of the businessman’s proposition and promptly moved him down.

(a) deception, forced
(b) naivete, turned
(c) potential, forced
(d) sophistry, turned
(e) No correction required

DOWNLOAD IBPS PO Question Papers PDF

Printed Study Material for IBPS PO Exam

IBPS PO (Probationary Officer) Exam 2018 Study Materials

IBPS / SBI Special TX: 
Subject: 
General: 

(Papers) IBPS PO Exam Paper - 2016 "Held on: 20-11-2016" ::COMPUTER KNOWLEDGE::

IBPS logo

(Papers) IBPS PO Exam Paper - 2016 "Held on: 20-11-2016"

::COMPUTER KNOWLEDGE::

1. Assembly is a _______ based low-level language replacing binary machine-code instructions, which are very hard to remember, it is the classic and uncontroversial example of a low level language.

(a) Memory
(b) High Level
(c) Key
(d) Mnemonic
(e) FORTRAN

2. China now has more of the world's fastest supercomputers than other countries. Which among the following is a Chinese super computer?

(a) BlueGene/Q system
(b) Cray XC30
(c) Shaheen II
(d) Fujitsu's K
(e) Tianhe-2

3. AT&T designed its first commercial modem, specifically for converting digital computer data to analog signals for transmission across its long distance network. What is the name of the modem?

(a) Telex
(b) Memex
(c) CompuServe
(d) Bell 103 dataset
(e) Dataphone

4. In Computer programming there is set of subroutine definitions, protocols, and tools for building software and applications. Which among the following is a term for sets of requirements that govern how one application can talk to another?

(a) UPS
(b) API
(c) CGI
(d) J2EE
(e) OLE

5. BSoDs can be caused by poorly written device drivers or malfunctioning hardware, such as faulty memory, power supply issues, overheating of components, or hardware running beyond its specification limits. Which color screen is displayed when encountered a BSOD Error?

(a) Red
(b) Grey
(c) Black
(d) Blue
(e) Green

6. Which among the following is a cloud computing platform and infrastructure created by Microsoft?

(a) Simple Storage Service
(b) Atmos
(c) Openstack Swift
(d) OceanStore
(e) Azure

7. After a credit card transaction has been authorized by the issuing bank by sending an authorization code to the merchant, the settlement stage of the process begins.

Which of the following is a system type used to process such statements?

(a) Multitasking
(b) Memory Processing
(c) Level Processing
(d) Batch Processing
(e) Online Processing

8. There is a network that can connect networks ranging from small location or area to a bigger range including public packet network and large corporate networks. That network's enterprise allows users to share access to applications, services and other centrally located resources. Its ability for a huge geographical access has transformed networking. Which among the following is that network?

(a) SAN
(b) CAN
(c) LAN
(d) WAN
(e) MAN

9. Which cloud is a cloud computing environment that uses a mix of on-premises, private cloud and third-party, public cloud services with orchestration between the two platforms and it is particularly valuable for dynamic or highly changeable workloads?

(a) Dynamic Cloud
(b) Advance Cloud
(c) Hybrid Cloud
(d) Sharing Cloud
(e) Combined Cloud

10. Which among the following is not an Object Oriented Programming Language?

(a) Python
(b) PASCAL
(c) Java
(d) C++
(e) Ruby

DOWNLOAD IBPS PO Question Papers PDF

DOWNLOAD IBPS CLERK Question Papers PDF

DOWNLOAD MORE BANK EXAMS E-BOOKS

Printed Study Material for IBPS PO Exam

IBPS PO (Probationary Officer) Exam 2018 Study Materials

IBPS / SBI Special TX: 
General: 

(Papers) IBPS PO Exam Paper - 2016 "Held on: 20-11-2016" ::GENERAL AWARENESS WITH REFERENCE TO SPECIAL BANKING::

IBPS logo

(Papers) IBPS PO Exam Paper - 2016 "Held on: 20-11-2016"

::GENERAL AWARENESS WITH REFERENCE TO SPECIAL BANKING::

1. The Nobel Peace Prize 2016 has been awarded to ______ .

(a) Juan Manuel Santos
(b) Oliver Hart
(c) Bengt Holmstrom
(d) Svetlana Alexievich
(e) Bob Dylan

2. Who among the following India's Paralympic athletes has won the title of Gold Javelin F46 in Rio 2016 Paralympic Games?

(a) Varun Singh Bhati
(b) Deepa Malik
(c) Amit Kumar Saroha
(d) Mariyappan Thangavelu
(e) Devendra Jhajaria

3. What is the Ramanujam committee constituted for?

(a) To keep tab on the child trafficking
(b) To check corruption in Banks
(c) To oversee the foreign trade
(d) To avoid obsolete laws
(e) To check border dispute with Bangladesh

4. How much per cent of the Indian population have access to internet in India in 2015?

(a) 20 percent
(b) 23 percent
(c) 29 percent
(d) 26 percent
(e) None of the above

5. According to the 2016 Jane's Defence Budgets Report, India's position among the global defence spender is _____________.

(a) Fourth
(b) Third
(c) Ninth
(d) Fifth
(e) Seventh

6. Which e-commerce website has partnered with National Highways Authority of India (NHAI) to enable cashless payments at all national and city toll plazas?

(a) Paytm
(b) Mobikwik
(c) Freecharge
(d) Oxigen
(e) UPI

7. Who among the following has been named as the 2016 ITF World Champions by the International Tennis Federation?

(a) Rafael Nadal
(b) Andy Murray
(c) Roger Federer
(d) Novak Dokovic
(e) None of the above

8. The National Green Tribunal (NGT) in December 2016 imposed a ban on the procurement, sale and use of glass powder coated 'manja' for flying kites. Who is the Chairman of NGT?

(a) Lokeshwar Singh Panta
(b) Jagdish Singh Khehar
(c) Dinesh Saxena
(d) Jasmine Sharma
(e) Swatanter Kumar

9. Which state government has enacted a stringent law called Protection of Internal Security Act to deal with the challenges of terrorism, insurgency, communalism and caste violence?

(a) Gujarat
(b) Uttar Pradesh
(c) Bihar
(d) Maharashtra
(e) Nagaland

10. India signed an MoU with which country for strengthening cooperation in the field of Tourism?

(a) Tajikistan
(b) Turkmenistan
(c) Uzbekistan (d) Kyrgyzstan
(e) UAE

11. Which of the following does not fall into the functions of Payments Banks?

(a) acceptance of demand deposits
(b) Internet banking
(c) remittance services
(d) lending services
(e) Sell mutual funds, insurance and pension products

12. Loan and Advances of a bank come under the category of ______________.

(a) Deposits
(b) Expenditure
(c) Liabilities
(d) Assets
(e) None of the above

13. If the rupee depreciates, how the exporters are affected?

(a) They are unaffected
(b) They are in loss
(c) They are in profit
(d) They get credit crunch
(e) None of these

14. The rate of inflation increases when the purchasing power of money___________ .

(a) Decreases
(b) Increases
(c) Stable
(d) Decrease just half
(e) None of these

15. The high-security banknote production facility which is credited with the design of the Rs 2000 banknote has been set up at _____________ .

(a) Nasik
(b) Dewas
(c) Mysuru
(d) Noida
(e) Kolkata

16. For the buying and selling of precious metals such as Gold and Silver, the market established is termed as ________ .

(a) Capital Market
(b) Money Market
(c) Bullion Market
(d) wholesale cash market
(e) None of the Above

17. To release wages under the Mahatma Gandhi National Rural Employment Guarantee Scheme direct and faster, an electronic fund management system has been launched by ____________.

(a) State Bank of Hyderabad
(b) State Bank of Travancore
(c) State Bank of India
(d) Oriental Bank Corporation
(e) Exim Bank

18. Which among the following is the slogan of Allahabad Bank?

(a) A tradition of trust
(b) World wise
(c) Honour your trust
(d) Invest with confidence
(e) Trusted family bank

19. Which among the following is the difference in value between a country's imports and exports?

(a) Balance of Trade
(b) Balance of Payment
(c) Balance of power
(d) Credit Balance
(e) None of the above

20. What is Cash Reserve Ratio?

(a) Deposits of banks in government securities
(b) Rate at which banks borrow funds from the RBI
(c) Deposits (as cash) which banks have to keep/maintain with the RBI
(d) Rate at which RBI borrows money from the banks
(e) Rate at which RBI borrows money from the government

Printed Study Material for IBPS PO Exam

DOWNLOAD IBPS PO Question Papers PDF

IBPS / SBI Special TX: 
General: 

(Papers) IBPS PO Exam Paper - 2016 "Held on: 16-10-2016" ::QUANTITATIVE APTITUDE::

IBPS logo


(Papers) IBPS PO Exam Paper - 2016 "Held on: 16-10-2016"

::QUANTITATIVE APTITUDE::


1. Vikram invests some money in three different schemes for 4 years, 8 years and 12 years at 10%, 15% and 20% Simple Interest respectively. At the completion of each scheme, he gets the same interest. The ratio of his investments is

(a) 6 : 2 : 1
(b) 5 : 2 : 1
(c) 5 : 2 : 3
(d) 5 : 2 : 7
(e) None of these

2. A sum of `3903 is divided between P and Q such that the share of P at the end of 8 years is equal to the share of Q after 10 years. Find the share of P if rate of interest is 4% compounded annually.

(a) 2012
(b) 2029
(c) 2028
(d) 2081
(e) None of these

3. Shopkeeper purchased some goods for `900 and sold onethird of the goods at a loss of what 12%, then at gain % should the remainder goods he sold to gain 18% profit on the whole transaction ?

(a) 31%
(b) 26%
(c) 33%
(d) 18%
(e) None of these

4. A truck covers a distance of 376 km at a certain speed in 8 hours. How much time would a car take at an average speed which is 18 kmph more than that of the speed of the truck to cover a distance which is 14 km more than that travelled by the truck ?

(a) 6 hours
(b) 5 hours
(c) 7 hours
(d) 8 hours
(e) 7.5 hours

5. Two trains are moving in opposite directions at 60 km/hr and 90 km/hr. Their lengths are 1.10 km and 0.9 km respectively. The time taken by the slower train to cross the faster train in seconds is:

(a) 58 sec
(b) 50 sec
(c) 48 sec
(d) 56 sec
(e) None of these

6. James' father was 30 years old when he was born. His mother's age was 24 when his sister who is 5 years younger to him, was born. What is the difference between the age of James' father and mother?

(a) 8
(b) 10
(c) 6
(d) 11
(e) 9

7. The average monthly expenditure of Mr. Ravi's family for the first three months is `2,750, for the next three months is `2,940 and for the last three months `3,150. If his family saves `4980 for nine months, find the average monthly income of the family for the 9 months?

(a) `3800
(b) `3500
(c) `3400
(d) `4200
(e) `4500

8. A and B undertake to complete a piece of work for Rupees 1200. A can do it in 8 days, B can do it in 12 days and with the help of C they complete the work in 4 days. Find the share of C?

(a) 100
(b) 200
(c) 300
(d) 400
(e) None of these

9. Three pipes A, B, and C can fill the tank in 10 hours, 20 hours and 40 hours respectively. In the beginning all of them are opened simultaneously. After 2 hours, tap C is closed and A and B are kept running. After the 4th hour, tap B is also closed. The remaining work is done by tap A alone. What is the percentage of the work done by tap A alone?

(a) 30 %
(b) 35 %
(c) 45 %
(d) 50 %
(e) None of these

10. In a school the number of boys and girls are in the ratio of 4:7. If the number of boys are increased by 25% and the number of girls are increased by 15%. What will be the new ratio of number of boys to that of girls?

(a) 100:131
(b) 100:151
(c) 100:161
(d) 100:181
(e) None of these

DIRECTIONS (11-15) : In these questions, a number series is given. Find out the missing number.

11. 279936, 46656, 7776, 1296, 216, ?

(a) 60
(b) 66
(c) 46
(d) 26
(e) 36

12. 139, 142, 133, 160, 79, ?

(a) 326
(b) 322
(c) 331
(d) 340
(e) 355

13. 164, 40, ?, 43, 188

(a) 224
(b) 68
(c) 90
(d) 176
(e) None of these

14. 96, 320, 800, 1600, ?

(a) 640
(b) 2666.7
(c) 2500
(d) 2400
(e) None of these

15. 6 4 6 12 22 ?

(a) 26
(b) 36
(c) 86
(d) 66
(e) 46

Printed Study Material for IBPS PO Exam

DOWNLOAD IBPS PO Question Papers PDF

IBPS / SBI Special TX: 
General: 

(Papers) IBPS PO Exam Paper - 2016 "Held on: 16-10-2016" ::REASONING ABILITY::

IBPS logo

(Papers) IBPS PO Exam Paper - 2016 "Held on: 16-10-2016"

::REASONING ABILITY::

1. What should come in place of question mark in the expression P>Q ? R < T < S so as to make the expressions P>R and S>Q always true?

(a) =
(b) >
(c) <
(d) ³
(e) None of these

2. What should come in place of question mark in the expression A = B >C ? D< E = F so as to make the expression F>C always true?

(a) >
(b) =
(c) ³
(d) £
(e) Both (b) and (d)

3. Statements: P = S, P < Q, R £ Q, R£ T
Conclusions : a) Q > S b) Q = T

(a) Only one follow
(b) Only two follow
(c) Neither follows
(d) Both follow
(e) Either follow

4. Statements: - A > N, K ³ N, K > M, R > M
Conclusions: - a) M = N b) R ³ A

(a) Only one follow
(b) Only two follow
(c) Neither follows
(d) Both follow
(e) Either follow

5. What should come in place of question mark to make B > D always true?

A = B > C ? D <E

(a) >
(b) <
(c) ³
(d) £
(e) both a and c

DIRECTIONS (6-10): Read the information carefully and answer the questions that follows:

(a) only I follows
(b) only II follows
(c) either I or II
(d) neither I nor II
(e) both I and II

6. Statements: Some pens are pencils. Some pencils are erasers. Some erasers are sharpeners. Some sharpeners are dusters.

Conclusions:

I. Some sharpeners are not pencils.
II. All dusters are pens.

7. Statements: All squares are circles. No circle is cone. Some cones are spheres. Some rectangles are circles.

Conclusions:

I. All rectangles being cones is a possibility
II. All rectangles being sphere is a possibility

8. Statements: All squares are circles. No circle is cone. Some cones are spheres. Some rectangles are circles.

Conclusions:

I. Some rectangles are not cones.
II. No square is cone

9. Statements: Some reds are greens. All greens are blues. All blues are oranges. No yellow is blue.

Conclusions:

I. No red is yellow.
II. Some oranges are greens

10. Statements: Some reds are greens. All greens are blues.
All blues are oranges. No yellow is blue.

Conclusions:

I. Some yellows are greens is a possibility.
II. All yellows being oranges is a possibility

11. Anil starts walking in east direction and after travelling some distance he took a right turn and then a left turn followed by another left turn. Now he again took a right turn and finally took a left turn. In which direction is anil walking.

(a) south
(b) north
(c) east
(d) west
(e) None of these

DIRECTIONS(Qs.12-15) Study the following information carefully to answer the given questions. Eight people A, B, C, D, E, F, G and H are sitting in a traight line with equal distances between each other,but not necessarily in the same order. Some of them are facing North and some of them are facing south.

• A sits at one of the extreme ends of the line. Only three people sit between A and G. E sits exactly between A and G.
• H sits third to the right of (e) B is an immediate neighbour of H and faces south. C sits second to the right of F. C is not an immediate neighbour of G.
• Immediate neighbour of G face opposite directions(i.e. if one neighbour faces North then the other neighbour faces south and Vice-Versa)
• A and D face the same direction as E(i.e if E faces north then A and D also face North and Vice-Versa). Both the immediate neighbours of E face south.

12. In the given arrangement, if two people come and sit to the immediate left of E, how many people will sit between F and C?

(a) Two
(b) Three
(c) Four
(d) More than four
(e) One

13. Who amongst the following sits third to the right of F?

(a) A
(b) B
(c) Other than those given as options
(d) D
(e) E

14. How many people face North as per the given arrangement?

(a) Two
(b) Three
(c) Four
(d) More than four
(e) One

15. Four of the following five are alike in a certain way based upon their seating arrangement and so form a group. Which of the following does not belong to the group?

(a) EC
(b) AF
(c) BF
(d) CG
(e) DG

16. Who amongst the following sits at the extreme end of the row?

(a) B
(b) C
(c) D
(d) E
(e) Other than those given as options

Printed Study Material for IBPS PO Exam

DOWNLOAD IBPS PO Question Papers PDF

IBPS / SBI Special TX: 
General: 

(Papers) IBPS PO Exam Paper - 2016 "Held on: 16-10-2016" ::ENGLISH LANGUAGE::

IBPS logo

(Papers) IBPS PO Exam Paper - 2016 "Held on: 16-10-2016"

::ENGLISH LANGUAGE::

DIRECTIONS (1-7): Read the following passage carefully and answer the questions given below it. Certain words are printed in bold to help you locate them while answering some of the questions.

Globalization is the objective trend of economic development in the world today, featured by free flow and optimized allocation of capital, technology, information and service in the global context. It is the inevitable result of the development of productive forces and advances of science and technology, especially the revolution of information technology since the1980s and 1990s. The influence of globalization on countries at different stages of development is entirely different. The "dividends" derived from globalization are not fairly distributed. The developed countries have apparent advantages in capital, technology, human resources and administrative expertise and in setting
the "rules of the game". They are usually the biggest beneficiaries of globalization. The developing countries on the other hand are on the whole in an unfavorably position.
Developing countries can obtain some foreign investment, advanced technologies and management expertise, but at the same time they are the most vulnerable to the negative impacts of globalization and lack the ability to effectively fend off and reduce the risks and pitfalls that come along with globalization. In the 1990s, especially in recent years, the gap between the North and the South has further widened. The economic sovereignty and economic security of the developing countries are confronted with enormous pressure and stern challenges. Some least-developed countries are even on the brink of being marginalized by globalization. Therefore, in participation of globalization, developing countries should always be on alert and try by all means to exploit the advantages and avoid all kinds of risk and harm. In the past 20-odd years, China has maintained an annual growth rate of over 9.3% on average. China is now the 6th largest economy and the 5th largest trading nation in the world. More than 200 million people have been lifted out of poverty. The above accomplishments were achieved against the backdrop of a volatile international situation. The reason why China was so successful in such a short period of time and in a constantly changing international environment is because China has found its own road of development i.e, to base what we do on the realities of China while sticking to the basic system of socialism, reforms should be carried out to solve the problems of incompatibility between the productive forces and
the relations of production, and between economic base and the superstructure, so as to achieve self-perfection of socialism. Every country is different from the other. It opens not only to developed countries, but also to developing countries, not only in economic field, but also in all areas of social development. At the same time, it is not a blind
opening, but a self-conscious one, not a disorganized opening but a systematic one. China's opening proceeds and deepens in a gradual and step by step fashion. It started from the 4 special economic zones, to coastal cities, then to capital cities of inland provinces and now it has reached an unprecedented stage of all-round opening demonstrated by China's accession to the World Trade Organization. During its opening-up, China paid special attention to give full play to its comparative advantages to actively conduct international cooperation and competition. For instance, China has fully exploited its advantages of low cost of labour to attract foreign investment and technology to push economic development and better efficiency and quality of economic growth. These measures have brought the Chinese economy increasingly integrated
with the world economy. China has learnt many lessons and accumulated rich experiences in dealing with globalisation from its practice of reform and opening-up. To adopt opening-up policy. It opens not only to developed countries, but also to developing countries, not only in economic field, but also in all areas of social development. At the same time, it is not a blind opening, but a self-conscious one, not a disorganized opening but a systematic one. China's opening proceeds and deepens in a gradual and step by step fashion. It started from the 4 special economic zones, to coastal cities, then to capital cities of inland provinces and now it has reached an unprecedented stage of all-round opening demonstrated by China's accession to the World Trade Organization. During its opening-up, China paid special attention to give full play to its comparative advantages to actively conduct international cooperation and competition. China's participation in Globalization is by no means a one-way street. When the world economic growth remains weak, China's economy is one of the few bright spots. As World Bank Report on Global Development Finance 2003 published in early April pointed out that China's fast growth "helped to drive the recovery in East Asia. Together with policy stimulus in other countries, China's performance lifted the region to growth of 6.7 % in 2002, up from 5.5% in 2001.China has also provided the world with the largest rising market. When more than 1.25 billion people become well-off, the demand on everything will be enormous. Just to give you an example, in the coming 10 years alone, China will import US$ 2 trillion of goods from the outside world. It goes without saying that we are also facing many challenges. For instance, with the accession to the WTO, China is faced with growing pressure from international competition. China's enterprises have to cope with fiercer competition not only at international market, but at home market as well. Nevertheless, opening the country to the outside world is China's basic and long-term state policy. China is committed to opening still wider to the outside world in an all-directional and multi-tiered way, with an even more active approach.

1. Why the "dividends" derived from globalization are not fairly distributed?

(a) Apprehension in embracing and seizing the opportunities presented by globalization
(b) Failing to adopt reforms to keep up with the steps of the changing world.
(c) Political disadvantage due to inactivity in the developing countries.
(d) Due to the lack of a just and equitable international economic order
(e) None of these.

2. What reason author has given for China’s achievement in such a short span of time?

(a) Signficant modifications in the basic system of socialism.
(b) Framing their models on Chinese characteristics rather than relying on plagiarism.
(c) As they gave much more impetus on advancement in technology, human resources and administrative expertise
(d) Their responsible approach as they remained vigilant against various risks, especially financial risks.
(e) None of these

3. Which of the following is the most suitable term for the nature of Chinese opening to the outside world?

(a) Progressive
(b) Self conscious
(c) Comprehensive
(d) Discerning
(e) Selective

4. How according to author China is contributing to World Economy?

(a) By giving the road development to other developing countries so that they can follow the same path.
(b) By providing a huge market to the World to supply the needs of billion uplifted Chinese population.
(c) By new advancements in technology and human resources.
(d) Focusing their attention to conduct international cooperation and competition.
(e) None of these.

5. Which of the following statement(s) is/ are true in context of the passage?

(i) Developing countries are raising their concern over China’s commitment to even more active approach towards opening to World.
(ii) Developing countries are usually the most active propellers of globalization.
(iii) China’s rise is a threat for the developing countries like America.

(a) Only (ii)
(b) Both (i) and (iii)
(c) Both (i) and (ii)
(d) Only (i).
(e) All of the above

6. What is the challenge that is faced by China?

(a) Need to fulfill the enormous demands of more than a billion Chinese people who have recently escaped from poverty.
(b) Adapting to the model adopted by the other countries so as to not get isolated.
(c) Growing pressure from the international market.
(d) Opening the country to the outside world
(e) None of these

7. Which of the following statement is false?

(a) Low cost of labour in China is key to attract foreign investment.
(b) Achieving self-perfection of socialism is a reason for China's incredible growth
(c) Developing country can suffer economically due to globalization.
(d) China is facing a much fiercer competition at home compared to international markets.
(e) None of these.

DIRECTIONS (8-12): Rearrange the following Six sentences (A), (B), (C), (D), (E) and (F) in the proper sequence to form a meaningful paragraph and then answer the questions given below.

A-It is the only country in the world that is carbon negative, which means it produces more oxygen than it consumes. B-Bhutan, sandwiched between the two most populous
nations on Earth, suffers for their sins. C-So far, so good. But then, two things happened. D-Carbon sinks, 70% forest cover, powered almost entirely by mountain streams—Bhutan is a poster child for green living. E-Glaciers are beginning to melt, flash floods and heavy rains—and even droughts—are common, and temperatures are climbing. F-One, India and China got richer.

8. Which of the following should be the First sentence of the given paragraph?

(a) E
(b) D
(c) C
(d) B
(e) A

9. Which of the following should be the Third sentence of the given paragraph?

(a) A
(b) B
(c) C
(d) D
(e) E

10. Which of the following should be the LAST sentence of the given paragraph?

(a) A
(b) C
(c) B
(d) D
(e) E

11. Which of the following should be the Fourth sentence of the given paragraph?

(a) F
(b) C
(c) B
(d) E
(e) D

12. Which of the following should be the Second sentence of the given paragraph?

(a) B
(b) D
(c) A
(d) C
(e) E

DIRECTIONS (Q.13-20): In the following passage, youhave abrief passage. In the following passage, some of the words have been left out. First read the passage over and try to understand what it is about. Then fill in the blanks with the help of the alternatives given. Big ideas come from tackling --83-- problems. When one is confronted with an overwhelming task, it’s pieces. Business jargon is full of phrases about that, like “pilot projects” and “low-hanging fruit.” They have their place, but in the repertory of management --84---, they should share their place with bold approaches to big challenges. Much of today’s most valuable management knowledge came from wrestling with such issues. The most complicated workplace in the middle of the last century was the automobile assembly plant. Drawn to its complexity where Peter F. Drucker, W. Edwards Deming, and Taiichi Ohno, among others. The work they and their disciples did, applied in industry after industry, is the basis of the best that we know about operations, managing people, innovation, organizational design, and much more. The most complex workplaces are tertiary care hospitals. These vast --85-- employ tens of thousands of people who, under one roof, do everything from neurosurgery to laundry. Each patient – that is to say, each “job” — calls on a different set of people with a different constellation of ---86---; even when the two patients have the same diagnosis, success may be --87-- differently. This is complexity of an order of magnitude greater
than automobile assembly, and anyone who --88--- hospitalized knows that management has thus far been unequal to the scope of task. The workers, managers, consultants, and scholars --89-- crack this nut will reshape industries and institutions just as ---90--- as Drucker, Deming, and Ohno did.

13. (a) Small
(b) big
(c) Irrelevant
(d) Buildings
(e) minor

14. (a) Weakness
(b) Strength
(c) Power
(d) practice
(e) symptom

15. (a) houses
(b) institute
(c) demagogue
(d) Forts
(e) enterprises

Printed Study Material for IBPS PO Exam

DOWNLOAD IBPS PO Question Papers PDF

IBPS / SBI Special TX: 
Subject: 
General: 

(Papers) IBPS PO Exam Paper - 2017-QUANTITATIVE APTITUDE

IBPS logo

(Papers) IBPS PO Exam Paper - 2017 


:: ::QUANTITATIVE APTITUDE:: ::


Note : There were 4 shifts on 7 Oct. In Some shifts 5 questions on quadratic were asked in place of 5 series.

Directions (36-41): Given below is the table which shows the total students in 4 different schools and percentage of students participating in Dance and Play in 4 different classes.

Class Total Students % of students participating
Dance Play
VI 500 15 08
VII 400 10 06
VIII 360 25 10
IX 250 10 12

Q36. What is the ratio of students participating in Dance from Class VII and IX together to the students participating in Play from class VI and VIII together?
(a) 43 : 53
(b) 65 : 76
(c) 44 : 57
(d) 63 : 71
(e) 62 : 77

Q37. What is the average of students in Play from all the classes?

IBPS logo

Q39. What is the sum of students who do not participate in dance and play from class VI and IX together?
(a) 720
(b) 480
(c) 620
(d) 580
(e) None of these

Q40. If 20% of students who participate in dance from class VI also participate in play then find the ratio of students from class VI who participated only in Dance to students participated only in play.
(a) 25 : 16
(b) 16 : 25
(c) 19 : 20
(d) 20 : 19
(e) 15 : 11

Q41. Students participating in Dance from class VII is what percent of students participating in play from class IX.

IBPS logo
Directions (42-46): What should come in place of question mark (?) in the following number series?
Q42. 3, 5, 15, 45, 113, ?

(a) 190
(b) 234
(c) 293
(d) 243
(e) 208

Q43. 17, 98, 26, 89, 35, ?
(a) 78
(b) 79
(c) 80
(d) 81
(e) 82

Q44. 3240, 540, 108, 27, ?, 4.5
(a) 12
(b) 7
(c) 9
(d) 8
(e) 6

Q45. 7, 4.5, 5.5, 12, 49, ? 
(a) 393
(b) 378
(c) 197
(d) 148
(e) 246

Q46. 2, 17, 89, 359, 1079, ?
(a) 2134
(b) 1081
(c) 2195
(d) 2159
(e) 1945

Directions (42-46): In the following questions two equations numbered (I) and (II) are given. You have to solve both equations and Give answer
(a) if x > y
(b) if x ≥ y
(c) if x < y
(d) if x ≤ y
(e) If x = y or the relationship cannot be established

Q42. (i) x2- 5x +6=0

(ii) 3y2+3y-18=0

Q43. (i) x2-11x+30=0

(ii) y2+y-20=0

Q44. (i) 2x2+2x-4=0

(ii) y2-5y+4=0

Q45. (i) 2x2+6x-16=0

(ii) y2-6y+5=0 

Q46. (i)  x2-4=0

(ii) y2-9y+20=0

Directions (47-52): Find out of the approximate value of ? in the following questions

IBPS logo
(a) 50
(b) 60
(c) 75
(d) 70
(e) 55

Q48. 55.01 – 345.02 ÷ 22.99 =2x?
(a) 20
(b) 25
(c) 22
(d) 15
(e) 18

IBPS logo
(a) 7
(b) 8
(c) 6
(d) 9
(e) 5

Q50. (111.99x5)  ÷ 14.02=11.002+?
(a) 34
(b) 19
(c) 39
(d) 29
(e) 38

Q51. 24.97% of 84.01 ÷ 6.995=?

(a) 3
(b) 4
(c) 5
(d) 7
(e) 6

IBPS logo

(a) 4950
(b) 4820
(c) 5550
(d) 5340
(e) 5260

Q53. Sum of the present ages of A, B, C and D is 76 years. After 7 years ratio of their ages is 7 : 6 : 5 : 8. What is C’s present age?
(a) 14
(b) 12
(c) 13
(d) 8
(e) 10

Q54. Sum of the length of two trains A and B is 660. The ratio of the speeds of A and B is 5 : 8. Ratio between time to cross an electric pole by A and B is 4 : 3. Find the difference in the length of two trains.
(a) 50
(b) 60
(c) 80
(d) 75
(e) 90

Q55. A mixture of milk and water in a jar contains 28 L milk and 8 L water. X L milk and X L water are mixed to form a mixture. If 40% of the new mixture is 20 L, then find the value of X.
(a) 7 L
(b) 8 L
(c) 6 L
(d) 5 L
(e) 9 L

Q56. A alone can do a work in 24 days. Time taken by A in completing 1/3 of work is equal to the time taken by B in completing 1/2 of the work. In what time A and B together will complete the work?
(a) 9 days
(b) 10 days
(c) 12 days
(d) days
(e) 48/8 days

Q57. Marked price of A is Rs. 1600 more than its cost price. When discount on A is 500 a profit of 25% is obtained. At what price should A be sold to obtain a 30% profit.
(a) 4800
(b) 5600
(c) 5400
(d) 5200
(e) None of these

Q58. The ratio of diameter and height of a right circular cylinder is 4 : 3. If diameter of the cylinder get reduced by 25% then its total surface area reduced to 318.5 square meter. What is the circumference of the base of the  cylinder.

(a) 28 π cm2
(b) 14 π cm2
(c) 35 π cm2
(d) 7 π cm2
(e) None of these

Q59. The ten’s digit of a three digit number is 3. If the digits of x are interchanged and the number thus formed is 396 more than the previous one. The sum of unit digit and hundred digit is 14, then what is the number?
(a) 480
(b) 539
(c) 593
(d) 935
(e) None of these

Q60. S1 is a series of 4 consecutive even numbers. If the sum of reciprocal of first two numbers of S1 is 11/60, then what is the reciprocal of third highest number of S1?

(a) 2/13
(b) 1/14
(c) 2/17
(d) 1/13
(e) None of these

Q61. A, B and C invested in a business in the ratio 6 : 8 : 9. If B invested for a period whose numerical value is 112.5% of B’s investment but A and C invested for one year. If profit of B at the end of the year is 16750 then what is the share of profit of C?
(a) 20225
(b) 22125
(c) 25225
(d) 25125
(e) 23125

Q62. A boat covers 18 km downstream in 3 hours. If speed of current is 331/3 % of its downstream speed. In what time will it cover a distance of 100 km upstream?
(a) 50 hour
(b) 40 hour
(c) 30 hour
(d) 60 hour
(e) 25 hour

Q63. The ratio of cost price to the selling price of an article is 5 : 6. If 20% discount is offered on marked price of the article then marked price is what percent more than cost price?

(a) 100/3 %
(b) 50%
(c) 40%
(d) 200/3%
(e) 60%

Q64. Ramesh has 20% savings with him from his monthly salary. If expenditure on clothing is 25% of overall expenditure and his total expenditure except clothing is 3600 then find his saving.
(a) 1000

(b) 1500
(c) 1600
(d) 1200
(e) 900

Direction (65-70): A bar graph is given below which shows two types of hats sold by seller A and seller B on five days.
IBPS logo

IBPS logo

(a) 110
(b) 114
(c) 116
(d) 118
(e) 120

Q68. What is the difference between the number of hats sold on Monday and Wednesday by B to the number of hats sold on Friday by both A & B together?
(a) 9
(b) 12
(c) 14
(d) 21
(e) 24

Q69. A sold 80% defective hats on Thursday and B sold 75% defective hats on the same day. Find the number of hats sold by A and B on Thursday that are not defective?
(a) 25
(b) 20
(c) 18
(d) 32
(e) 40

Q70. Find the ratio of number of hats sold by A on Tuesday & Friday together to number of hats sold by B on same days.
(a) 25 : 23
(b) 23 : 25
(c) 21 : 25
(d) 25 : 21
(e) 18 : 17

General: 

(Papers) IBPS PO Exam Paper - 2017-REASONING ABILITY

IBPS logo

(Papers) IBPS PO Exam Paper - 2017 


:: REASONING ABILITY ::


Direction (1-5): Read the following information carefully and answer the questions given below.

L, M, N, O, P, Q and R are seven employees who are working in the same company. They attend meeting in different department viz; administrative, Security, Finance and HR department on different days from Monday to Sunday but not necessarily in the same order. One employee attends only one meeting and only one meeting is held on each day. There are two employees who attend meeting in administrative, security, HR department and only one employee attends meeting in Finance department.

L attends meeting on Thursday. There are two persons who attend meeting between L and the person who attends meeting in HR department. There are three persons who attend meeting between the persons who attend meeting in Administrative department and the one who attends meeting in Finance department. The one who attends meeting in administrative department attends before the one who attends in finance department. The one who attends meeting in finance department does not attend on Saturday. The number of persons who attend meeting between L and the one who attend meeting in finance department is same as the number of persons who attend meeting between O and the one who attends meeting in security department. The one who attends meeting in security department attend before O.O does not attend meeting in HR department. Q attends meeting on the day immediately before the day on which L attends meeting. O does not attend meeting on the day just after the day on which L attends meeting.The number of persons who attend meeting between L and P is same as the number of persons who attend meeting between L and R.P attends meeting in one of the day before the day on which R attends meeting.N attends meeting in administrative department. R does not attend meeting in security  department.

Q1. Who among the following person attend meeting on Friday?
(a) O
(b) M
(c) P
(d) N
(e) R

Q2. Which of the following combinations of “Person – Day” is true with respect to the given arrangement?
(a) R – Friday
(b) M-Saturday
(c) Q– Thursday
(d) P – Friday
(e) P – Tuesday

Q3. L attends meeting in which of the following department?
(a) Security
(b) HR
(c) Administrative
(d) Finance
(e) Either (a) or (b)

Q4. In this arrangement, Q is related to Monday, L is related to Security then N is related to?
(a) Thursday
(b) Wednesday
(c) None of the given options is true.
(d) HR
(e)Sunday

Q5. How many persons attend meeting between P and O?
(a) 3
(b) 4
(c) 2
(d) 1
(e)None of these

OR

Direction (1-5): Read the following information carefully and answer the questions: Seven persons A, B, C, D, E, F ,G like even colours i.e. Yellow, White, Red, Orange, Blue, Grey, and Black. They visit in a mall on different days starting from Monday to Sunday 

(i) A visits one of the day after Thursday.
(ii) Only 4 people are in between A and B.
(iii) The one who likes Red colour visits immediately after B.
(iv) Only one person visits between Red and Blue.
(v) The one who likes white colour visits before one of days on which day C visits.
(vi) The one who likes white colour does not visit on Monday. D visits before E.
(vii) Only 1 person is in between D and E. D likes Yellow colour.
(viii) The number of persons who visit between A and the who likes Blue color is one less than the number of person who visit between B and C.
(ix) Neither G nor F likes Black. Neither G nor F likes white colour.
(x) G and E do not visit on Saturday and G doesn’t like grey colour.

Q1. The one who visits just before F, likes which of the following colour?
(a) White
(b) Red
(c) Orange
(d) Blue
(e) None of these

Q2. Who amongst the following visits on Sunday?
(a) B
(b) The one who likes red colour
(c) A
(d) None of these
(e) The one who likes grey

Q3. Which of the following is true according to the given arrangement?
(a) D likes white colour
(b) F visits on Wednesday
(c) C visits just after G
(d) A likes grey colour
(e) None of these

Q4. F visits on which of the following day?
(a) Monday
(b) Tuesday
(c) None of these
(d) Saturday
(e) Sunday

Q5. A likes which of the following colour?
(a) White
(b) Yellow
(c) Red
(d) Black
(e) None of these

IBPS / SBI Special TX: 
Subject: 
General: 
STUDY KITS: 

(Papers) IBPS PO Exam Paper - 2017 - ENGLISH LANGUAGE

IBPS logo

(Papers) IBPS PO Exam Paper - 2017


:: ENGLISH LANGUAGE ::


Direction (01-10): Read the following passage carefully and answer the questions given below it. Certain words are given in bold to help you locate them while answering some of the questions.

IBPS / SBI Special TX: 
Subject: 
General: 

IBPS-RRBs Officer Scale-I Previous Year Exam Paper - 2014, "Reasoning"

IBPS-RRBs Officer Scale-1 Previous Year Exam Paper - 2014

Subject : Reasoning

Based on Memory

Directions (Q. Nos. 1-6) These questions consist of a question and two statements numbered I and II given below it. You have to decide whether the data given in the statements are sufficient to answer the question. Read both the statements and choose the most appropriate option.

1. Among friends M, N, O, P, Q and R, who is the second heaviest?

I. O is heavier than only two friends. P is heavier than Q but lighter than N. R is the heaviest.
II. M is lighter than only two friends. N is heavier than O but lighter than R. P is heavier than only Q.

(a) The data either in statement I alone or statement II alone are sufficient to answer the question.
(b) The data in both statements I and II together are necessary to answer the question.
(c) The data in statements I alone are sufficient to answer the question, while the data in statement II alone are not sufficient to answer the question.
(d) The data in statement II alone are sufficient to answer the question, while the data in statement I alone are not sufficient to answer the question.
(e) The data even in both statements I and II together are not sufficient to answer the question.

2. Among six people E, F, G, H, I, J standing around a circle facing the centre, what is the position of G with respect to F?

I. E stands second to the right of G. Only one person stands between E and I. F is an immediate neighbour of G.
II. Only two people sit between G and H. H is an immediate neighbour of both I and E. F is not an immediate neighbour of I.

(a) The data in statement II alone are sufficient to answer the question, while the data in statement I alone are not sufficient to answer the question.
(b) The data either in statement I alone or statement II alone are sufficient to answer the question.
(c) The data in statement I alone are sufficient to answer the question, while the data in statement II alone are not sufficient to answer the qusetion.
(d) The data even in both statements I and II together are not sufficient to answer the qustion.
(e) The data in both statements I and II together are necessary to answer the quesiton.

3. In which month of the year did Rahul go abroad for his vacation?

I. Rahul correctly remembers that he went for his vacation in the first half of the year.
II. Rahul’s friend correctly remembers that he went for his vacation after 31st March and before 1st May.

(a) The data either in statement I alone or statement II alone are sufficient to answer the question.
(b) The data in both statements I and II together are necessary to answer the question.
(c) The data in statement I alone are sufficient to answer the question, while the data in statement II alone are not sufficient to answer the question.
(d) The data in statement II alone are sufficient to answer the question, while the data in statement I alone are not sufficient to answer the question.
(e) The data even in both statements I and II together are not sufficient to answer the question.

4. How many marks has Suman scoredd in a test having 20 maximum marks.

I. Suman scored two-digit even marks in the test.
II. Suman scored more than 14 but less than 18 marks in the test.

(a) The data either in statement I alone or statement II alone are sufficient to answer the question.
(b) The data in both statements I and II together are necessary to answer the question.
(c) The data in statement I alone are sufficient I alone are sufficient to answer the question while the data in statement II alone are not sufficient to answer the question.
(d) The data in statement II alone are sufficient to answer the question, while the data in statement I alone are not sufficient to answer the question.
(e) The data even in both statements I and II together are not sufficient to answer the question.

5. Who among P, Q, R, S and T is the tallest?

I. P is taller than Q, T is not the tallest.
II. R is taller than P, S is not the tallest.

(a) The data either in statement I alone or statement II alone are sufficient to answer the question.
(b) The data in both statements I and II together are necessary to answer the question.
(c) The data in statement II alone are sufficient to answer the question, while the data in statement II alone are not sufficient to answer the question.
(d) The data in statement II alone are sufficient to answer the question, while the data in statement I alone are not sufficient to answer the question.
(e) The data even in both statements I and II together are not sufficient to answer the question.

(e-Book) IBPS RRB Officer Scale-I Previous Year Papers

(e-Book) IBPS RRB Office Assistants Previous Year Papers

NEW! IBPS RRB (Regional Rural Bank) Study Notes

6. What is the age of Rakesh in a group of four members?

IBPS / SBI Special TX: 
Bank/Organisation: 
Subject: 
General: 

(Paper) IBPS RRB CWE (IV) Office Assistant (Multipurpose) Exam Paper - 2015 "held on 12th September 2015" (Reasoning)

https://bankexamportal.com/sites/default/files/IBPS-RRB-LOGO.jpeg

(Paper) IBPS RRB CWE (IV) Office Assistant (Multipurpose) Exam Paper - 2015 "held on 12th September 2015"


Memory Based

Directions (1-5): In the questions, relationship between different elements is shown in the statements. The statements are followed by conclusions. Study the conclusions based on the given statements and select the appropriate answer.

Give answer

(a) If either conclusion I or II is true
(b) If only conclusion I is true
(c) It both conclusion I and II are true
(d) If neither conclusion I nor II is true
(e) If only conclusion II is true

1. Statements

Conclusions

I. C > D
II. H < S

2. Statements

Conclusions

I. E = P
II. P > E

3. Statements

S > J = T < O; T < P

Conclusions

I. S > O
II. P < S

4. Statements

Conclusions

I. R < A
II. V > G

5. Statements

(e-Book) IBPS RRB Officer Scale-I Previous Year Papers

(e-Book) IBPS RRB Office Assistants Previous Year Papers

Download IBPS RRB Officer (Scale-1) Papers PDF

IBPS / SBI Special TX: 
General: 

IBPS Clerk Prelims Online Question Paper - 2018 (Reasoning)

IBPS-CLERK Papers PDF


IBPS Clerk Prelims Online Question Paper - 2018 (Reasoning)


 Subject : Reasoning


Direction (66 – 69): Read the given information and answer the question that follows. 7 people P, Q, R, S, T, V, and W give a test starting from Monday to Sunday. 4 people give the test between R and S. P gives the test on Wednesday. No person gives the test between P and Q. There are 3 people who give the test between T and V. T gives the test before V. W does not give the test on Sunday. At least 2 people give the test before S

66. How many people attempt the test after R?
A. 1 
B. 5
C. 3 
D. 4
E. None

67. Who gives the test on Sunday?
A. P 
B. R
C. S 
D. Q
E. V

68. Which of the following statement is false?
A. Q gives the test on Thursday.
B. P gives the test before S.
C. Three people give the test between W and R.
D. V gives the test after S.
E. All of the above are false.

69. W gives the test in which of the following days?
A. Monday 
B. Tuesday
C. Thursday 
D. Friday
E. Saturday

70. How many pairs of letters are there in the word UNSETTLED, each of which has as many letters between (in both forward and backward direction) them in the word as they have between them in the English alphabet?
A. One 
B. Two
C. Three 
D. More than three
E. None

Direction (71 – 74): The question consists of some statements followed by some conclusions. Consider the statements to be true even if they are in variance with the commonly known facts. Read all the conclusions and decide which of the given conclusions logically follow from the given statements and accordingly mark your answer.

71. Statements:
All Bikes are Car
Some Car are Truck
Conclusions:
I. Some Bike are Truck
II. No Truck is Car
A. Only conclusion I follow.
B. Neither conclusion I nor II follows.
C. Both conclusions I and II follow.
D. Either conclusion I or II follows.
E. Only conclusion II follows.

72. Statements:
Some Toffee are eclairs
All eclairs are dairy-milk.
Some dairy-milk are choc-bar.
Conclusions:
I. At least some eclairs are dairymilk.
II. No Toffee is choc-bar.
A. Only conclusion I follows.
B. Neither conclusion I nor II follows.
C. Both conclusions I and II follow.
D. Either conclusion I or II follows.
E. Only conclusion II follows.

73. Statements:
Some box are Table
No table is board.
Some board are wall.
Conclusions:
I. Some board are box.
II. No box is board.
A. Only conclusion I follow.
B. Neither conclusion I nor II follows.
C. Both conclusions I and II follow.
D. Either conclusion I or II follows.
E. Only conclusion II follows.

74. Statements:
Some Toffee are eclairs
All eclairs are dairy-milk.
Some dairy-milk are choc-bar.
Conclusions:
I. Some choc-bar being toffee is a possibility.
II. No Toffee is dairy-milk.
A. Only conclusion I follow.
B. Neither conclusion I nor II follows.
C. Both conclusions I and II follow.
D. Either conclusion I or II follows.
E. Only conclusion II follows.

Direction (75 – 79): Study the following information to answer the given questions. Twelve people are sitting in two parallel rows containing six people each, in such a way that there is an equal distance between adjacent persons. In row-1 A, B, C, D, E and F are seated and all of them are facing south. In row-2 P, Q, R, S, T and V are seated and all of them are facing north. Therefore, in the given seating arrangement each member seated in a row faces another member of the 11 other row. V sits third to the right of S. S faces F and F does not sit at any of the extreme ends of the line. D sits third to the right of C. R faces C. The one who is facing E sits third to the right of P. B and P do not sit at the extreme ends of the line. T is not an immediate neighbour of V and A is not an immediate neighbour of C. R does not sit at the extreme end.

75. Who amongst the following faces D?
A. T 
B. P
C. Q 
D. R
E. None of these

76. Who amongst the following represent the people sitting at the extreme ends of the rows?
A. R, F 
B. T, A
C. D, R 
D. C, Q
E. S, A

77. Four of the following five are alike in a certain way and thus form a group. Which is the one that does not belong to that group?
A. B-T 
B. A-Q
C. C-S 
D. F-P
E. D-R

78. Four of the following five are alike in a certain way and thus form a group. Which is the one that does not belong to that group?
A. D 
B. S
C. V 
D. T
E. A

79. How many persons are seated between R and T?
A. ONE 
B. TWO
C. THREE 
D. FOUR
E. None of these

Direction (80 – 84): Study the information given below and answer the questions based on it. Eight friends, P, Q, R, S, T, Y, V, and W are sitting around a square table in such a way that four of them sit at four corners of the square, while four sit in the middle of each of the four sides. The ones who sit at the four corners face the centre, while those who sit in the middle of the sides face outside.
* P who faces the centre sits third to the right of V.
* T, who faces the centre, is not an immediate neighbour of V.
* Only one person sits between V and W.
* S sits second to the right of Q, Q faces the centre.
* R is not an immediate neighbour of P.

80. Who sits second to the left of Q?
A. V 
B. P
C. T 
D. Y
E. W

81. What is the position of T with respect to V?
A. Fourth to the left
B. Second to the left
C. Third to the left
D. Third to the right
E. Second to the right

82. Four of the following five are alike in a certain way and so form a group. Which is the one that does not belong to that group?
A. R 
B. W
C. V 
D. S
E. Y

83. Which of the following will come in place of the question mark (?) based upon the given seating arrangement? WP TR QW RS ?
A. YT 
B. VY
C. VQ 
D. PY
E. QV

84. Which of the following is true regarding R?
A. R is an immediate neighbor of V
B. R faces the centre
C. R sits exactly between T and S
D. Q sits third to left of R
E. None is true

Direction (85 – 87): Read the given information and answer the question that follows. E is 10 Km to the west of I. C is 5 Km to the south of E. A is 15 km to the east of C. F is 10 km to the west of H. B is 5 km to the west of A. H is 5 km to the north of E. D is 15 km to the west of C. G is 5 km to the west of F.

85. Which of the following are not sitting in a straight line?
A. HEC 
B. CBA
C. IFC 
D. GFH
E. DCB

86. What is the direction of G with respect to A?
A. South
B. North
C. North – West
D. South – West
E. South – East

87. B is in which direction with respect to C and at what distance?
A. 10 km, West
B. 5 km, East
C. 10km, East
D. 5 km, West
E. none of the above

Direction (88 – 90): Study the information given below and answer the questions based on it. Six persons M, N, O, P, Q and R have different heights. N is taller than P. M is taller than R but shorter than P. At least one person is shorter than R. The number of people taller than Q is same as shorter than O. Q is taller than M.
88. How many persons are taller than Q?
A. 0 
B. 1
C. 2 
D. 3
E. 4

89. Who among the following is the shortest in the group?
A. O 
B. R
C. P 
D. N
E. Q

90. Who among the following is taller than N?
A. P 
B. R
C. M 
D. Q
E. O

Direction (91 – 95): Read the given information and answer the question that follows. Seven friends are visiting seven different cities namely, Jhansi, Delhi, Ooty, Shimla, Amritsar, Lucknow, and Bhuvneshwar but not necessarily in the same order. The one who visits Jhansi is immediately above the one who visits Lucknow. There are only two persons between the one who visits Amritsar and the one who visits Jhansi. The one who visits Ooty is above Amritsar but not immediately above the one who visits Amritsar. Only three persons are between the one who visits Shimla and the one who visits Ooty. The one who visits Bhuvneshwar is immediately above the one who visits Shimla. Only one person is there between the one who visits Delhi and the one who visits Bhuvneshwar.

91. How many people are visiting between the one who visits Ooty and the one who visits Bhuvneshwar?
A. One 
B. Two
C. Three 
D. Four
E. None

92. Which of the following city is visited immediately before Delhi?
A. Ooty 
B. Amritsar
C. Delhi 
D. Bhuvneshwar
E. None

93. Which of the following cities are there between Jhansi and Amritsar?
A. Shimla, Bhuvneshwar
B. Ooty, Shimla
C. Lucknow, Delhi
D. Lucknow, Ooty
E. None of these

94. Which of the following condition is correct regarding Amritsar with respect to Bhuvneshwar?
A. There is one city between Bhuvneshwar and the one who visits Amritsar
B. Bhuvneshwar is immediately above the one who visits Amritsar
C. All the above are true
D. The one who visits Amritsar is the person immediately above Bhuvneshwar
E. None of these

95. Which of the following cities is there exactly between Delhi and Jhansi?
A. Amritsar 
B. Ooty
C. Shimla 
D. Lucknow
E. None of these

Direction (96 – 100): The questions are based on the five three- letter word given below.

96. If all the letters in the words are arranged in reverse alphabetical order and then after all the word are arranged in alphabetical order from left to right, then which of the following word is 2 nd from the right end thus formed?
A. PLG 
B. UND
C. TNK 
D. NIC
E. None of these

97. In each word every consonant is changed to the previous letter in English alphabetical series then how many words have at least single vowels?
A. One 
B. Two
C. Three  
D. Four
E. None of these

98. In each of the word if we interchange the first letter with third, then the first letter with the second one, then which of the following is the third word according to English dictionary (answer in terms of original word)?
A. NKT 
B. BEA
C. DNU 
D. ICN
E. None of these

99. If we interchange second letter with third and then first letter with third letter then what will be the value if we add all the place values of first letter in each word thus formed?
A. 30 
B. 35
C. 40 
D. 45
E. None of these

100.If we add all the letters place value in each word then which of the following word number is completely divisible by 9?
A. BEA 
B. NKT
C. DNU 
D. ICN
E. None of these

DOWNLOAD IBPS CLERK Question Papers PDF

DOWNLOAD IBPS PO Question Papers PDF

DOWNLOAD MORE BANK EXAMS E-BOOKS

Printed Study Material for IBPS Clerk Exam

IBPS / SBI Special TX: 
Subject: 
General: 
STUDY KITS: 

IBPS Clerk Prelims Online Question Paper - 2018 (Numerical Ability)

IBPS-CLERK Papers PDF


IBPS Clerk Prelims Online Question Paper - 2018 (Numerical Ability)


 Subject : Numerical Ability






46. What was the ratio of item sold by P on Saturday & items sold by Q on Sunday?
A. 9:11 
B. 10:13
C. 8:11 
D. 9:13
E. None of them

47. Items sold by P on Tuesday is what percent of item sold by S on Sunday?
A. 140% 
B. 130%
C. 120% 
D. 125%
E. None of these

48. What is the average of the items sold by P, Q and S on Monday, Tuesday and Saturday respectively?
A. 33 
B. 35
C. 28 
D. 30
E. None

49. Find the ratio of item sold by P on Saturday and Sunday to item sold by S on Monday & Tuesday?
A. 17:16 
B. 14:11
C. 16:17 
D. 17:15
E. None of these

50. Q sold certain items on Wednesday which was 80% of the total items sold by Q on Saturday and Sunday both. find the items sold by Q on Wednesday.
A. 86 
B. 66
C. 76 
D. 56
E. None of these

51. A sum becomes 5 times of itself on simple interest in 5 years. What is the rate of interest?
A. 100% 
B. 80%
C. 40% 
D. 20%
E. 60%

52. A passenger train that runs at the speed of 78 km/h leaves the station 8 hours after the goods train left and overtakes it is 5 hours. What is the speed of goods train?
A. 15 km/h 
B. 30 km/h
C. 60 km/h 
D. 13 km/h
E. 72 km/h

53. In a class of 40 students, the number of boys is 4 more than the number of girls. The average weight of the entire class is 48.4 KGs. If the average weight of girls is 44 KGs, what is the average weight of boys?
A. 50 kg 
B. 51 kg
C. 52 kg 
D. 53 kg
E. None of these

54. Sum of the speed of the boat in upstream and downstream is 36km/h. The speed of the stream is 3km/h. find the time taken to cover the 52.5 km upstream, assume the speed of the boat in still water is constant throughout.
A. 3 hours 
B. 4 hours
C. 3.5 hours 
D. 4.5 hrs
E. None of these

55. A man invested 15% of his monthly income in LIC and the remaining amount gave to his mother. Her mother spent 10% of it in household expenditure and thus she saved Rs.30,600, then find the monthly salary of the man.
A. 41500 
B. 42800
C. 43400 
D. 40000
E. None of these

56. In a city, 68% population is literate in which the ratio of the number of males and the number of females is 11:6. And if the number of illiterate males to the number of illiterate females ratio is 3:1. Find the ratio of literate female to illiterate female.
A. 1:3 
B. 2:1
C. 3:1 
D. 2:3
E. None of these

57. A is 1.5 times more efficient than B and C is two times efficient than A. A and B take 7(1/2) days to complete the work. How many days will B & C take to complete the work together?
A. 4(1/6) days 
B. 5(2/3) days
C. 5(5/6) days 
D. 3(5/6) days
E. None of these

58. In an exam if 7 marks given for right Answer and 4 marks deducted for wrong Answer and total marks given to a student is 263, then how many right Answer given by student if total attempts is 58?
A. 44 
B. 45
C. 42 D. 40
E. None of these

59. The difference between Circumference of circle A and diameter is 90 cm. If Radius of Circle B is 7 cm less than A then find area of Circle B?
A. 616 cm2 
B. 308 cm2
C. 77 cm2 
D. 49 cm2
E. None of these

DOWNLOAD IBPS CLERK Question Papers PDF

DOWNLOAD IBPS PO Question Papers PDF

DOWNLOAD MORE BANK EXAMS E-BOOKS

Printed Study Material for IBPS Clerk Exam

IBPS / SBI Special TX: 
General: 
STUDY KITS: 

IBPS Clerk Prelims Online Question Paper - 2019 ( Reasoning Ability)

IBPS-CLERK Papers PDF


IBPS Clerk Prelims Online Question Paper - 2019 ( Reasoning Ability)


 Subject: Reasoning Ability


Direction (66 – 70) : Study the following information carefully and answer the questions given below. Twelve people are sitting in two parallel rows containing six people in each row. In row 1 - B, C, D, E, F and G are sitting and all of them are facing north and in row 2 - Q, R, S, T, U and V are sitting and all of them are facing south but not necessarily in the same order. In the given arrangement, each member seated in the row faces another member of the other row. D sits at one of the extreme ends of the row. Two people sit between B and D. The one who faces B sits second to the left of Q. Three people sit between Q and S. The one who faces S sits to the immediate right of G. As many people sit to the left of G as to the left of V. Two people sit between T and V. R sits at one of the positions to the right of U. The one who faces U sits third to the right of E. F sits at one of the positions to the right of C.

66. Who sits fourth to the left of F?
A. G
B. C
C. B
D. E

67. How many people sit between S and R?
A. One
B. Two
C. Three
D. Four

68.Who faces C?
A. R
B. Q
C. V
D. U

69.Which of the following statements is true?
A. Three people sit between B and D
B. V sits opposite to G
C. T and E sit at the extreme ends of the row
D. R and U are immediate neighbours

70. Four of the following five are alike in a certain way, find the odd one out.
A. Q
B. T
C. D
D. R

71. How many such pairs of letters are there in the word ‘DECLARING’ each of which has as many letters between them in the word as in the English alphabet (both forward and backward)?
A. One
B. Two
C. Three
D. More than three

Direction (72 – 74) : In each of the questions below are given two/three statements followed by two conclusions. You have to take the given statements to be true even if they seem to be at variance from commonly known facts. Read all the conclusions and then decide which of the given conclusions logically follow from the given statements disregarding commonly known facts.

72. Statements:
All ladders are stairs.
Only a few stairs are ways.
All ways are elevators.
Conclusions:
I. No elevators are ladders.
II. Some ladders are elevators.
A. If only conclusion I follows
B. If only conclusion II follows
C. If either conclusion I or II follows
D. If neither conclusion I nor II follows

73. Statements:
A few computers are type-writers.
Only a few type-writers are laptops.
Conclusions:
I. All laptops being type-writers is a
possibility.
II. All computers being laptop is a
possibility.
A. If only conclusion I follows
B. If only conclusion II follows
C. If either conclusion I or II follows
D. If both conclusions I and II follow

74. Statements:
Few trucks are papers.
All papers are magazines.
Conclusions:
I. Some trucks are definitely not
magazines.
II. Some magazines are trucks.
A. If only conclusion I follows
B. If only conclusion II follows
C. If either conclusion I or II follows
D. If neither conclusion I nor II follows

Direction (75 – 79) : Study the following information and answer the given questions carefully. Point B is 10m to the West of point A. Point C is 15m to the South of point B. Point D is 15m to the North of point A. Point E is 15m to the East of point D. Point C is 15m to the West of point F. Point G is 10m to the East of point C. Point H is 15m to the North of point F. Point R is 3m to the East of point E.

75. What is the distance and direction of point H with respect to F?
A. 15 m to the South
B. 10 m to the North
C. 15 m to the North
D. 10 m to the South

76. What is the direction of G with respect to R?
A. North East
B. North
C. South
D. South West

77. Which of the following do not represent straight line?
A. CGF
B. GAD
C. BAH
D. FHE

78. Find the odd one out.
A. RF
B. DC
C. DR
D. RB

79. Which of the following statements is true?
A. Point D is to the South of point A
B. Point H is to the East of point B
C. Point H and G lie on the same line
D. Point F is to the West of point G

80. Four of the following five are alike in some way, find the odd one out.
A. SNM
B. HCB
C. RML
D. WTS

Direction (81 – 82) : In each of the questions, relationships between some elements are shown in the statements. These statements are followed by two conclusions numbered I and II. Read the statements and give the answer.

81. Statements:
S ≥ T = A ≤ N < D
Conclusions:
I. D > T
II. S < N
A. Only conclusion I is true
B. Only conclusion II is true
C. Either conclusion I or conclusion II is true
D. Neither conclusion I nor conclusion II is true

82. Statements:
B ≤ R < K = A; T < R
Conclusions:
I. T < K
II. A > B
A. Only conclusion I is true
B. Only conclusion II is true
C. Either conclusion I or conclusion II is true
D. Both conclusion I and II are true

Direction (83 – 87) : Study the information given below and answer the questions based on it. Eight People A, B, C, D, P, Q, R and S born in eight different months namely, January, March, April, May, June, August, October and December but not necessarily in the same order. A was born in a month after August. Only four people born between A and B. Only two people born between B and C. The number of people born between A and C is the same as the number of people born between C and D. P was born in one of the months before D. Only two people born between P and Q. R was born before S but after Q. 83. C was born in which of the following months?
A. June
B. April
C. December
D. March

84. Who was born in the month of May?
A. S
B. D
C. Q
D. B

85. The number of people born before R as after __.
A. C
B. D
C. B
D. Q

86. Which of the following statements is true?
A. B was born in the month of April
B. Only three people born between S and Q
C. B was born in the month of January
D. R was born immediately before D

87. If P is related to D, B is related to Q, in the same way C is related to which of the following?
A. A
B. R
C. B
D. S

Direction (88 – 90) : Study the information given below and answer the questions based on it. Five boxes A, B, C, D and E of different weights are arranged in a row from top to bottom. The weight of Box A is more than the weight of Box B but Box A is lighter than Box C. Box D is lighter than Box C but not the lightest. Box C is not the heaviest among them. As many boxes are heavier than Box A as the number of boxes lighter than Box C. The weight of the second lightest box is 15 kg.

88. Who among the following is the third lightest?
A. E
B. C
C. B
D. D

89. If the weight of the Box C is 18kg, then what can be the weight of Box D?
A. 16kg
B. 10kg
C. 20kg
D. 12kg

90. How many boxes are lighter than Box E?
A. 4
B. 2
C. 3
D. None

Direction (91 – 95) : Study the following data carefully and answer the questions accordingly. Six people A, B, C, D, E, and F are sitting around a circular table facing outside. C is not sitting second to the left of F. E sits 12 third to the right of A. B sits third to the right of F. C and A are not neighbors. B and A are neighbors.

91. Who is sitting third to the right of C?
A. A
B. F
C. E
D. D

92. Who is sitting opposite to E?
A. F
B. A
C. D
D. B

93. Who among the following are sitting opposite to each other?
A. F, C
B. C, A
C. E, D
D. F, B

94. Who is sitting immediate right of F?
A. B
B. E
C. A
D. D

95. Find the odd one from the following.
A. B, D
B. C, F
C. A, E
D. F, A

Direction (96 – 100) : Read the following information carefully to answer the questions given below. In a certain code language, ‘training is always necessary’ is written as ‘god asb byo mnp’, ‘teacher perception always right’ is written as 'mnp den coh hen', 'teacher can stop training’ is written as ‘byo coh vyo rho’ ‘stop bad perception necessary’ is written as ‘den puk asb vyo’.

96. What is the code used for ‘right’?
A. mnp
B. hen
C. den
D. coh

97. ‘Puk’ is the code used for which of the following?
A. stop
B. perception
C. bad
D. necessary

98. ‘hen god puk’ would mean?
A. is right training
B. bad is right
C. training is bad
D. training right bad

99. ‘teacher is always linguist’ would be written as
A. god coh pa mnp
B. byo asb mnp coh
C. mnp god coh vyo
D. god vyo den asb

100. Which word is represented by ‘byo’?
A. necessary
B. stop
C. Training
D. perception

DOWNLOAD IBPS CLERK Question Papers PDF

DOWNLOAD IBPS PO Question Papers PDF

DOWNLOAD MORE BANK EXAMS E-BOOKS

Printed Study Material for IBPS Clerk Exam

IBPS / SBI Special TX: 
Subject: 
General: 
STUDY KITS: 

IBPS Clerk Prelims Online Question Paper - 2019 (Numerical Ability)

IBPS-CLERK Papers PDF


IBPS Clerk Prelims Online Question Paper - 2019 (Numerical Ability)


 Subject: Numerical Ability







41. 8 years hence the sum of A’s and B’s age will be 70 years. 4 years ago, the ratio between sum of ages of A and B together and C’s age was 2 : 1. What is the present age of C?
A. 23 years
B. 27 years
C. 32 years
D. 37 years

42. A and B invested Rs. 4800 and Rs. 7200 respectively. B invested for two more months than A. If the total annual profit is Rs. 5750 and the share of A in profit is Rs. 2000, then for how many months did B invest?
A. 8 months
B. 9 months
C. 6 months
D. 10 months

43. A boat travels 28.8 km in upstream direction in 2 hours 24 minutes. If the ratio of upstream speed to downstream speed is 2 : 3. Find the speed of boat in still water.
A. 12 km/hr
B. 15 km/hr
C. 16 km/hr
D. 10 km/hr

44. ‘X + 10’ is 20% more than ‘Y – 8’. If X is 22 more than Y, then find the value of Y.
A. 208
B. 188
C. 202
D. 148

45. From a jar containing milk and water solution, 20% of the mixture is taken out and same quantity of water was added to the mixture. If the respective ratio between resultant quantity of milk and water is 32 : 15. Find the ratio of the initial amount of milk and water?
A. 20 : 7
B. 40 : 7
C. 10 : 7
D. 5 : 14



Direction (46 – 50) : In the bar graph, number of cakes sold by 5 bakeries – A, B, C, D and E in the week I is given.
46. Number of cakes sold by bakery C is approximately what percent of the number of cakes sold by bakery E?
A. 40%
B. 35%
C. 45%
D. 55%

47. Find the ratio between number of cakes sold by bakery B to number of cake sold by bakery E.
A. 5 : 8
B. 5 : 4
C. 3 : 8
D. 5 : 16

48. Out of the total number of cakes sold by bakery A, 1/6th were designer cake, then what is the number of non-designer cakes sold by bakery A?
A. 32
B. 16
C. 30
D. 15

49. What is average number of cakes sold by bakeries A and D?
A. 44
B. 48
C. 36
D. 42

50. Total number of cakes sold by bakeries C and E together increases by 20% in week II compared to week I, then what was the total number of cakes sold by bakeries C and E in week II?
A. 96
B. 108
C. 112
D. 99



Direction (56 – 63) : Find the wrong term in the given series:
56. 1, 3, 6, 24, 120, 720, 5040
A. 1
B. 3
C. 6
D. 120

57. 5, 6, 13, 40, 161, 803, 4837
A. 161
B. 4837
C. 13
D. 803

58. 420, 429, 426, 437, 432, 443, 438
A. 438
B. 443
C. 420
D. 429

59. 50, 46, 55, 39, 63, 28, 77
A. 50
B. 46
C. 39
D. 63

60. 10, 12, 9, 14, 6, 16, 1
A. 10
B. 9
C. 1
D. 16

61. If a boat travels 29 km in upstream direction in 5 hours and the speed of the stream is 2.2 km/hr, then the time taken by boat to travel a distance 81.6 km in downstream direction is
A. 8 hours
B. 9 hours
C. 6 hours
D. 5 hours

62. Area of a square is 784 sq. metre. If the diameter of a circle is equal to the side of the square, then what is the circumference of the circle?
A. 110 metre
B. 130 metre
C. 105 metre
D. 88 metre

63. A shopkeeper bought X kg of a tea at the rate of Rs. 56/kg and 32 kg of a tea at the rate of Rs. 91/kg. If he made a profit of 25% by selling the mixture at the rate of Rs. 105/kg, then find the value of
X.
A. 6
B. 10
C. 12
D. 8

Direction (64 – 65) : Read the information carefully and answer the following questions on the basis of the information given below.

DOWNLOAD IBPS CLERK Question Papers PDF

DOWNLOAD IBPS PO Question Papers PDF

DOWNLOAD MORE BANK EXAMS E-BOOKS

Printed Study Material for IBPS Clerk Exam

IBPS / SBI Special TX: 
General: 
STUDY KITS: 

IBPS Clerk Prelims Online Question Paper - 2019 (English Language)

IBPS-CLERK Papers PDF


IBPS Clerk Prelims Online Question Paper - 2019 (English Language)


 Subject : English Language


Direction (1 – 5) : In the following question, a sentence with four words printed in bold is given. These words are numbered as (1), (2), (3) and (4). One of these four words may either be misspelt or inappropriate in the context of the sentence. Find out the word(s) that is wrongly spelt or inappropriate and mark the relevant option as your answer. If all the words printed in bold are correct, mark option (E), i.e. ‘All are correct’, as the answer.

1. When we reach home after travelling(1) for weaks, (2) the home is the same but something (3) in our mind has changed and that changes everything.(4)
A. 4
B. 1
C. 2
D. 3

2. Many people believe (1) that it is better to lead a slow life with limited (2) pleasures (3) than the mindless (4) rat race of the city.
A. 4
B. 3
C. 2
D. All are correct

3. With rice (A) in online consumer base, many online retailers (B) are finding it difficult to fulfil (C) all the consumer demands in a short (D) time.
A. C
B. D
C. A
D. B

4. Given the pace (A) at which technology is developing (B) today, it is clever (C) that our world (D) will work completely different in just a few days.
A. D
B. C
C. A
D. B

5. Participating (A) in various competitions (B) since childhood, helps students develop (C) a spirrit (D) of sportsmanship.
A. B
B. A
C. D
D. C

Direction (6 – 8) : A sentence with one blank is given, indicating that something has been omitted. Choose the word that best fits in the blank making the sentence
grammatically correct and meaningful.

6. The infrastructure projects of the city are falling behind schedule thus increasing the ______ of the residents.
A. Inconvenience
B. Safety
C. Working
D. Stressful

7. There has been a significant rise in the price of certain goods and commodities recently, making them ______for the common man.
A. Price
B. Progressing
C. Increasing
D. Unaffordable

8. It is vital that children, young people and all of us have access to stories which give us the knowledge, empathy and understanding we need to __________
life.
A. Engage
B. Negotiate
C. Overlook
D. Developing

Direction: A sentence with one blank is given, indicating that something has been omitted. Choose the word that best fits in the blank making the sentence
grammatically correct and meaningful.

9. Tigers are large, solitary predators who owe their entire existence to being able to ___________ detection by prey and sneak close before attacking.
A. Practice
B. Indulge
C. Avoid
D. Advance

Direction (10 – 11) : Read the sentence to find out whether there is an error in it. The error, if any, will be in one part of the sentence. The number corresponding to
that part will be your answer. If the given sentence is correct as it is, mark the answer as ‘No error’. Ignore the errors of punctuation, if any.

10. Having supportive parents (1)/ help massive in (2)/ the positive development and (3)/growth of a child. (4)
A. 1
B. 2
C. 3
D. 4

11. The reports suggested that (1)/ some animals is (2)/ likely to be extinct (3)/ in the near future. (4)
A. 1
B. 2
C. 3
D. 4

Direction: Read the sentence to find out whether there is an error in it. The error, if any, will be in one part of the sentence. The number corresponding to that part
will be your answer. If the given sentence is correct as it is, mark the answer as ‘No error’. Ignore the errors of punctuation, if any.

12. As per the new design (1)/ all appliances inside (2)/ the building will (3)/ be power by solar energy. (4)
A. 1
B. 2
C. 3
D. 4

Direction: Read the sentence to find out whether there is an error in it. The error, if any, will be in one part of the sentence. The number corresponding to that part
will be your answer. If the given sentence is correct as it is, mark the answer as ‘No error’. Ignore the errors of punctuation, if any.

13. A person driven by (1)/ greed and envious, loses (2)/ the ability to see (3)/ things as they are. (4)
A. 1
B. 2
C. 3
D. 4

Direction (14 – 20) : Read the given passage carefully and answer the questions that follow. Certain words are printed in bold to help you locate them while answering some of these.

Why do we travel? It's not the flying I mind – I will always be awed by the physics that gets a fat metal bird into the upper troposphere. The rest of the journey, however, can feel like a tedious lesson in the ills of modernity, from the pre-dawn X-ray screening to the sad airport malls peddling crappy souvenirs. It's globalisation in a nutshell, and it sucks. And yet here we are, herded in ever greater numbers on to planes that stay the same size. Sometimes we travel because we have to. Sometimes we travel because we want to, because the annoyances of the airport are outweighed by the visceral thrill of being someplace new. Because work is stressful and our blood pressure is too high and we need a vacation. Because home is boring. Because the flights were on sale. Travel, in other words, is a basic human desire. We're a migratory species, even if our migrations are powered by jet fuel and Chicken McNuggets. But here's my question: is this collective urge to travel – to put some distance between ourselves and everything we know – still a worthwhile compulsion? Or is it like the taste for saturated fat: one of those instincts we should have left behind in the Pleistocene epoch? Because if travel is just about fun, then I think the new security measures at airports have killed it. The good news, at least for those of you reading this while stuck on a tarmac, is that pleasure is not the only consolation of travel. In fact, several new science papers suggest that getting away – and it doesn't even matter where you're going – is an essential habit of effective thinking. It's not about a holiday, or relaxation: it's about the taxing act itself, putting some miles between home and wherever you happen to spend the night. The reason such travels are mentally useful involves a quirk of cognition, in which problems that feel "close" – and the closeness can be physical, temporal or even emotional – get contemplated in a more concrete manner. As a result, when we think about things that are nearby, our thoughts are _____(A)_____, bound by a more limited set of associations. While this habit can be helpful – it allows us to focus on the facts at hand – it also inhibits our imagination. When we escape from the place we spend most of our time, the mind is suddenly made aware of all those errant ideas we'd suppressed. We start thinking about obscure possibilities that never would have occurred to us if we'd stayed back on the farm. Furthermore, this more relaxed sort of cognition comes with practical advantages, especially when we're trying to solve difficult problems.

14. Which of the following is true about travelling?
A. We travel because we need a break from stressful work.
B. The idea of travelling is enhanced by the security arrangements at the airport.
C. It is on rare occasions that we derive pleasure through travelling.
D. All of these

15. Which of the following is true with respect to the passage?
A. An escape from a regular environment allows us to think about possibilities, which we would otherwise never explore.
B. Apart from deriving pleasure, we can also develop a habit of effective thinking through travelling.
C. The author finds the idea of an aeroplane flying in the troposphere absurd.
A. Only A
B. Both A & B
C. Only C
D. Both A & C

16. With which of the following is the author most likely to agree?
A. Travelling allows us to be aware of the ideas suppressed within us.
B. Globalisation has benefitted us in many ways.
C. Travelling bridges the distance between ourselves and everything we know.
A. Only B
B. Only A
C. Only C
D. Both A & B

17. Which of the following is the MOST SIMILAR in meaning to the given word as used in the passage?
Taxing
A. Strenuous
B. Burdensome
C. All of these
D. Stressful

18. Which of the following is the MOST SIMILAR in meaning to the given word as used in the passage?
Obscure
A. Uncertain
B. Likely
C. Clear
D. Difficult

19. Which of the given statements can be concluded from the following statement given in the passage?
"It's not about a holiday, or relaxation: it's about the taxing act itself, putting some miles between home and wherever you happen to spend the night"
A. A stressful work forces one to travel away for some time.
B. A change from the existing surroundings is a habit ingrained in very nature of human species.
C. Human derives pleasure from continuous travelling.
D. Humans like flying over the skies in confined spaces.

20. Which of the following words can be used to fill the blank labelled (A) in the passage?
A. Liberated
B. Accessible
C. Constricted
D. Reluctant

Direction (21 – 25) : Rearrange the following six sentences (A), (B), (C), (D) and (E) in a proper sequence to form a meaningful paragraph, then answer the questions that follow.
(A) These two conclusions by the trust are based on empirical data gathered from surveys carried out in many parts of the country.
(B) The National Literacy Trust has noted that becoming a lifetime reader is based on developing a deep love of reading.
(C) So, the big challenge for teachers is not simply getting students to read – it's getting them to enjoy it too.
(D) In other words, teachers do not need to motivate students to read too much but encourage reading for pleasure.
(E) It further notes that this is especially true for young readers as motivation to read decreases with age and if children do not enjoy reading when they are young,
then they are unlikely to do so when they get older.

21. Which of the following will be the fourth statement after rearrangement?
A. B
B. E
C. D
D. C

22. Which of the following will be the fifth statement after rearrangement?
A. D
B. C
C. A
D. E

23. Which of the following will be the first statement after rearrangement?
A. C
B. E
C. D
D. B

24. Which of the following will be the second statement after rearrangement?
A. A
B. B
C. C
D. E

25. Which of the following will be the third statement after rearrangement?
A. A
B. B
C. D
D. E

Direction (26 – 30) : In the following passage, there are blanks each of which has been numbered. These numbers correspond to the question numbers. Against each question, five words have been suggested, one of which would fill the blank appropriately. Mark the suitable word as the answer. 
Amy's school needed to have a fundraiser so they could (###Q1###) money to buy more computers. Amy knew that many of her classmates' families were from (###Q2###) countries around the world. They had many special traditions, spoke many different languages and ate many different types of foods. Amy had a (###Q3###) idea for a fundraiser! She suggested that every student could bring in their favourite dish and hold an ethnic dinner night. She knew parents and members of the community would be (###Q4###) to pay money in order to try foods from all over the world! Amy with the help of her friends organised the event. The fundraiser was a great (###Q5###). Everyone enjoyed seeing, smelling and tasting foods from so many different cultures.

26. Find the appropriate word in each case.
A. Waste
B. Income
C. Earn
D. Occupy

27. Find the appropriate word in each case.
A. Alike
B. Variety
C. Moderate
D. Different

28. Find the appropriate word in each case.
A. Vague
B. Brilliant
C. Elaborate
D. Dumb

29. Find the appropriate word in each case.
A. Agree
B. Outwit
C. Concern
D. Glad

30. Find the appropriate word in each case.
A. Success
B. Celebrity
C. Disaster
D. Variation

DOWNLOAD IBPS CLERK Question Papers PDF

DOWNLOAD IBPS PO Question Papers PDF

DOWNLOAD MORE BANK EXAMS E-BOOKS

Printed Study Material for IBPS Clerk Exam

IBPS / SBI Special TX: 
Jobs: 
General: 
STUDY KITS: 

IBPS Clerk Prelims Online Question Paper - 2018 (English Language)

IBPS-CLERK Papers PDF


IBPS Clerk Prelims Online Question Paper - 2018 (English Language)


 Subject : English Language


Direction (1 – 5) : Read the given passage carefully and answer the questions that follow. Certain words are printed in bold to help you locate them while answering some of these. A decade ago, working from home may have seemed like a distant dream. Today, it's the future of work. With millennials the driving force behind this shift in the workforce, remote work was dismissed as another "annoying" millennial trait. As more companies adopt telecommuting practices, the benefits are very clear. No longer is remote work about working on a beach, it's a complete design in how we approach the way we work. Here are the benefits and reasons why to embrace telecommuting and remote work. It is estimated that employers in the US lose $1.8 trillion a year in productivity. From distractions like water cooler gossip to excessive commuting, health problems and more. Workers are finding it harder than ever to hit maximum productivity in a traditional office work environment. The solution? Telecommuting. According to the State of Work Productivity Report, 65% of full-time employees think a remote work schedule would increase productivity. This is backed up by more than twothirds of managers reporting an increase in overall productivity from their remote employees. Where do telecommuters find this extra boost of productivity? With none of the distractions from a traditional office setting, telecommuting drives up employee efficiency. It allows workers to retain more of their time in the day and adjust to their personal mental and physical wellbeing needs that optimize productivity. Removing something as simple as a twenty-minute commute to work can make all world of difference. If you are ill, telecommuting allows one to recover faster without being forced to be in the office. It also improves the impact on our overall health. As a diabetic, health is the number one priority. If our health is not in good condition, then it is inevitable that our productivity will also decline. The average person in the U.K. spends 60-80 minutes getting to and from work. With the drive to reduce our carbon footprint, telecommuting is an excellent way for your company to go green and improve your employees’ general well-being. In a report published by the Royal Society for Public Health in the UK, it found that 55% of people felt more stressed as a result of their commute. Snacking habits also increased and with less free time available, the report also found that workers were leading less active and healthy lifestyles. In a 2014 study by PGI, a leading provider of software services, it found that 80% of remote workers reported higher morale, 82% said it helped lower their stress levels, and 69% reported lower absenteeism. https://www.forbes.com

1. Why does the author support the idea of “working from home”?
I. It allows the older generation to remain in the workforce.
II. It enhances the productivity of the workers as it saves them from social distractions.
III. It cuts down the compensation cost of the employer.
A. Only I
B. Only II
C. Both I and II
D. Both II and III
E. All of these

2. How does working through telecommunication ensure the personal wellbeing of the employees?
I. By providing them time to cater to their psychological wellness.
II. It facilitates speedy recovery from any physical illness.
III. It compromises with the overall professional productivity of an employee.
A. Both II and III
B. Both I and II
C. Only II
D. Only I
E. All of these

3. Which of the following statement is false with reference to the passage?
A. The productivity of people who are not feeling well declines if they attend office.
B. Telecommuting can be a way to reduce the carbon footprint.
C. People working in the traditional office environment are highly unproductive.
D. Work from home helps lower stress level and absenteeism.
E. None of the above.

4. What does the author mean by the phrase, “No longer is remote work about working on a beach…”?
A. Work is conducted seriously while working from home.
B. Earlier, the idea of remote work included working from holiday locations.
C. Earlier, the concept of remote working lacked popularity.
D. Remote working provides the scope of holidaying while managing work.
E. None of the above

5. Which of the following is the MOST SIMILAR in meaning to the given word?
Optimize
A. Malign
B. Accurate
C. Eliminate
D. Approve
E. None of the above

Direction (6 – 10): Rearrange the following five sentences (A), (B), (C), (D), and (E) in a proper sequence to form a meaningful paragraph; then answer the questions given below them. A) And people like the Indian President and Prime Minister get 3 cents and 1 cent respectively. B) Sachin Tendulkar, arguably India’s best-ever cricket player, earns some $30 a minute. C) India’s highest-paid CEO, Mukesh Ambani of Reliance Industries, gets $10, and celluloid superstar Amitabh Bachchan, $8. D) That makes it tempting to assume that the business of cricket must be huge and thriving, too. E) But even if you ignore the numbers, the conclusion is that India’s cricket stars are handsomely paid

6. Which of the following should be the FIRST sentence after rearrangement?
A. B
B. D
C. A
D. C
E. E

7. Which of the following should be the THIRD sentence after rearrangement?
A. B
B. D
C. A
D. C
E. E

8. Which of the following should be the FIFTH sentence after rearrangement?
A. B
B. D
C. A
D. C
E. E

9. Which of the following should be the SECOND sentence after rearrangement?
A. B
B. D
C. A
D. C
E. E

10. Which of the following should be the FOURTH sentence after rearrangement?
A. B
B. D
C. A
D. C
E. E

Direction: In the given question, a part of the sentence is printed in bold. Below the sentence alternatives to the bold part are given at (A), (B), (C) and (D) which may help improve the sentence. Choose the correct alternative. In case the given sentence is correct, your answer is (E), i.e., 'No correction required'.

11. The government has proposed changes in planning laws to ensure that nature conservation agencies and planning authorities allow of the directives.
A. abide by
B. ask for
C. aim at
D. act for
E. No correction required.

Direction: In the given question, a part of the sentence is printed in bold. Below the sentence alternatives to the bold part are given at (A), (B), 4 (C) and (D) which may help improve the sentence. Choose the correct alternative. In case the given sentence is correct, your answer is (E), i.e., 'No correction required'. 

12. Odysseus thought that it was still possible that the Greek army could muscle in the crisis in the war.
A. muscle into
B. muddle up
C. muddle through
D. muscle out
E. No correction required.

Direction: In the given question, a part of the sentence is printed in bold. Below the sentence alternatives to the bold part are given at (A), (B), (C) and (D) which may help improve the sentence. Choose the correct alternative. In case the given sentence is correct, your answer is (E), i.e., 'No correction required'.

13. By the end of this year, California will completely book in from its 5-yearlong recession.
A. bounce into
B. bounce back
C. box in
D. box up
E. No correction required.
 

Direction: In the given question, a part of the sentence is printed in bold. Below the sentence alternatives to the bold part are given at (A), (B), (C) and (D) which may help improve the sentence. Choose the correct alternative. In case the given sentence is correct, your answer is (E), i.e., 'No correction required'.

14. The book was libellous, so the publishers had to call upon all copies of it from the bookshops.
A. call for 
B. call forth
C. call in 
D. call on
E. No correction required

Direction: In the given question, a part of the sentence is printed in bold. Below the sentence alternatives to the bold part are given at (A), (B), (C) and (D) which may help improve the sentence. Choose the correct alternative. In case the given sentence is correct, your answer is (E), i.e., 'No correction required'.

15. You can no longer brush up the argument that the system is a sort of economic straitjacket, one that is becoming increasingly onerous.
A. bring up 
B. bring over
C. bring back 
D. brush off
E. No correction required

Direction: In this question, two columns I and II and three sentences are given, which are divided into two parts. Column I (A, B and C) consists of the first half of each sentence and Column II (D, E and F) consists of the second half of each sentence. Match column I with column II, so that the sentences formed are both meaningful and grammatically correct. Choose the relevant option as your answer

16. I.
A) There seems to be no drop in the form
B) The hatred that both countries have for each other
C) In his salad days, my grandfather was
II.
D) seems to have amplified after the recent incident.
E) Have been the best motorcycle in Germany.
F) Of Ronaldo after the World Cup.
A. C-E and A-D
B. B-D
C. A-F, C-D and B-E
D. A-F and B-D
E. C-D

Direction: In this question, two columns I and II and three sentences are given, which are divided into two parts. Column I (A, B and C) consists of the first half of each sentence and Column II (D, E and F) consists of the second half of each sentence. Match column I with column II, so that the sentences formed are both meaningful and grammatically correct. Choose the relevant option as your answer.

17. I.
A) Both companies were targetting 5
B) My uncle had a dog, who
C) Strategic missiles represents a logical step
II.
D) From a car company.
E) Students from college who were in their early twenties.
F) In the attempt to attack enemy forces at a distance.
A. A-D and C-E
B. C-E and B-F
C. A-E
D. A-E, B-D and C-F
E. C-D

Direction: In this question, two columns I and II and three sentences are given, which are divided into two parts. Column I (A, B and C) consists of the first half of each sentence and Column II (D, E and F) consists of the second half of each sentence. Match column I with column II, so that the sentences formed are both meaningful and grammatically correct. Choose the relevant option as your answer. 18. I.
A) Selfish politicians should never make it
B) Being rich doesn’t guarantee you
C) There are apps that calculate the range of your electric car,
II.
D) Usually in the form of a circle on a map showing the approximate range.
E) Goes to the backseat of the car.
F) There was still and attempt made.
A. C-D
B. B-D and A-F
C. A-E, B-D and C-F
D. C-F
E. B-D and C-E

Direction: In this question, two columns I and II and three sentences are given, which are divided into two parts. Column I (A, B and C) consists of the first half of each sentence and Column II (D, E and F) consists of the second half of each sentence. Match column I with column II, so that the sentences formed are both meaningful and grammatically correct. Choose the relevant option as your answer.
19. I.
A) The entire experience of using electric
B) The dog was chasing a squirrel
C) Despite all the efforts
II.
D) Of having to live with that.
E) Someone needs to tell him.
F) mobility should be much more convincing.
A. A-D
B. C-F and B-E
C. A-D, B-E and C-F
D. A-F
E. A-E and C-D

Direction: In this question, two columns I and II and three sentences are given, which are divided into two parts. Column I (A, B and C) consists of the first half of each sentence and Column II (D, E and F) consists of the second half of each sentence. Match column I with column II, so that the sentences formed are both meaningful and grammatically correct. Choose the relevant option as your answer.

20. I.
A) Some men are not interested in
B) There were ample reasons why you
C) There is no statute that specifically states that texting
II.
D) can be treated on a par with criminal negligence.
E) Playing football as they find it a risky sport.
F) It was meant to be.
A. A-D, B-E and C-F
B. B-D and A-F
C. A-E and C-D
D. C-D
E. A-E

Direction: A sentence with one blank is given, indicating that something has been omitted. Choose the word that best fits the blank appropriately.

21. Usually, when the local festival is a week away, the number of customers in the shop ________.
A. Will have increased
B. Has increasing
C. Will be increased
D. Increases
E. Increased

Direction: A sentence with one blank is given, indicating that something has been omitted. Choose the word that best fits the blank appropriately.

22. You will have to undergo a _____ of tests before the interview.
A. large 
B. series
C. design 
D. booklet
E. delegation

Direction: A sentence with one blank is given, indicating that something has been omitted. Choose the word that best fits the blank appropriately.

23. One has to be very _______ during the monsoons as the roads become slippery.
A. Smart 
B. Cautious
C. Presentable 
D. Prepare
E. Healthy

Direction: A sentence with one blank is given, indicating that something has been omitted. Choose the word that best fits the blank appropriately.

24. The judge used his _________ in settling the matter.
A. Know 
B. Tactics
C. Discretion 
D. Brains
E. Bias

Direction: A sentence with one blank is given, indicating that something has been omitted. Choose the word that best fits the blank appropriately. 

25. The Board members could not arrive at a unanimous _______ on the issue.
A. Judgement 
B. Progress
C. Policy 
D. Dissent
E. Action

Direction: In the given sentence, some words are printed in bold, one of which may be wrongly spelt or inappropriate in the context. Choose that word as the answer. If all the words printed in bold are correct, mark E, i.e. ‘All are correct’ as the answer.

26. The protagonist wanted to unearth the marvellous power which magnetized the needle and supervised a thousand celesial observations.
A. unearth 
B. marvellous
C. supervised 
D. celesial
E. All are correct

Direction: In the given sentence, some words are printed in bold, one of which may be wrongly spelt or inappropriate in the context. Choose that word as the answer. If all the words printed in bold are correct, mark E, i.e. ‘All are correct’ as the answer.

27. We can ensure more business investment in the region if we have laws which guarantee property rites.
A. business 
B. region
C. guarantee 
D. rites
E. All are correct

Direction: In the given sentence, some words are printed in bold, one of which may be wrongly spelt or inappropriate in the context. Choose that word as the answer. If all the words printed in bold are correct, mark E, i.e. ‘All are correct’ as the answer.

28. Unless more information is collected, the police will not reveal the details of the altercution between the arresting officer and the suspect.
A. information
B. reveal
C. altercution
D. suspect
E. All are correct

Direction: In the given sentence, some words are printed in bold, one of which may be wrongly spelt or inappropriate in the context. Choose that word as the answer. If all the words printed in bold are correct, mark E, i.e. ‘All are correct’ as the answer.

29. Considering the fact that Indians have outsed the non-natives almost always, it is not flawed to state that the nation will remain integrated till the kingdom come.
A. fact 
B. outsed
C. flawed 
D. conjointly
E. All are correct

Direction: In the given sentence, some words are printed in bold, one of which may be wrongly spelt or inappropriate in the context. Choose that word as the answer. If all the words printed in bold are correct, mark E, i.e. ‘All are correct’ as the answer.

30. He was famous not only for his interest in schemes for the allevation of poverty in Moscow, but also as the founder of new churches and monasteries.
A. schemes
B. allevation
C. founder
D. monasteries
E. All are correc

DOWNLOAD IBPS CLERK Question Papers PDF

DOWNLOAD IBPS PO Question Papers PDF

DOWNLOAD MORE BANK EXAMS E-BOOKS

Printed Study Material for IBPS Clerk Exam

IBPS / SBI Special TX: 
Subject: 
General: 
STUDY KITS: 

IBPS (PO) Previous Year Exam Paper - 2013 "Quantitative Aptitude"

IBPS logo


IBPS (PO) Previous Year Exam Paper - 2013

Subject: Quantitative Aptitude


Directions (Q. Nos. 91-95) In each of the following questions, a question is followed by information given in three Statements I, II and III. You have to study the question along with the statements and decide the information given in which of the statement(s) is necessary to answer the question.

91. In how many days 10 women can finish the work?

I. 10 men finish the work in 6 days.
II. 10 women and 10 men finish the work in days.
III. If 10 men work 3 days and after that 10 women are deployed to work for men, the rest work is finished in 4 days.

(1) I and II
(2) Any two of three
(3) I and III
(4) II and III
(5) None of these

92. What is the present age of Sabir?

I. The present age of Sabir is half of his father’s age.
II. After five years the ratio of ages of Sabir and his father is 6:11.
III. Sabir is younger to his brother by five years.

(1) 1 and II
(2) I and III
(3) II and III
(4) All of these
(5) Cannot be determined

93. What is two digit number?

I. The difference between the number and the number formed by interchanging the digit is 27.
II. The difference between two digits is 3.
III. The digit at unit’s place is less than that at ten’s place by 3.

(1) 1 and II
(2) I and either II or III
(3) I and III
(4) All of these
(5) None of these

94. What is the rate of interest Percent per annum?

I. An amount doubles itself in 5 yr on simple interest.
II. Difference between the compound interest and the simple interest earned on a certain amount in two years is Rs400.
III. Simple interest earned per annum is Rs2000.

(1) Only I
(2) II and III
(3) Any two of three
(4) I or II and III
(5) Only I or II and III

95. What is the cost of flooring the rectangular hall?

I. Length and the breadth of the hall are in the ratio of 3:2.
II. Length of the hall is 48 m and cost of flooring is Rs850 per sq m.
III. Perimeter of the hall is 160 m and cost of flooring is Rs850 per sq m.
(1) I and II (2) I and III
(3) Only III
(4) I and either II or III
(5) Any two of the three

96. If the numerator of a fraction is increased by 20% and the denominator is increased by 25%, the fraction obtained is . What was the original fraction?

(1) 5/7
(2) 4/7
(3) 3/8
(4) Cannot be determined
(5) None of the three

97. If the positions of the digits of a two-digit number are interchanged, the number obtained is smaller than the original number by 27. If the digits of the number are in the ratio of 1 : 2, what is the original number?

(1) 36 (2) 63
(3) 48
(4) Cannot be determined
(5) None of these

98. One of the angles of a quadrilateral is thrice the smaller angle of a parallelogram. The respective ratio between the adjacent angles of the parallelogram is 4:5. Remaining three angles of the quadrilateral are in ratio 4:11:9 respectively. What is the sum of the largest and the smallest angles of the quadrilateral?

(1) 255°
(2) 260°
(3) 265°
(4) 270°
(5) None of these

99. An aeroplane flies with an average speed of 756 km/h. A helicopter takes 48 h to cover twice the distance covered by aeroplane in 9 h. How much distance will the helicopter cover in 18 h? (Assuming that flights are non-stop and moving with uniform speed.)

(1) 5010 km
(2) 4875 km
(3) 5760 km
(4) 5103 km
(5) None of these

Directions (Q. Nos. 105-109) In the following number series, a wrong number is given. Find out the wrong number.

105. 29, 37, 21,43 ,13, 53, 5

(1) 37
(2) 53
(3) 13
(4) 21
(5) 43

106. 600, 125, 30, 13, 7.2, 6.44, 6.288

(1) 6
(2) 10
(3) 15
(4) 12
(5) None of these

107. 80, 42, 24, 13.5, 8.75, 6.375, 5.1875

(1) 8.75
(2) 13.5
(3) 24
(4) 6.375
(5) 42

108. 10, 8, 13, 35, 135, 671, 4007

(1) 8
(2) 671
(3) 135
(4) 13
(5) 35

109. 150, 290, 560, 1120, 2140, 4230, 8400

(1) 2140
(2) 560
(3) 1120
(4) 4230
(5) 290

Directions (Q.Nos. 110-114) These questions are based on the following data. The distribution of appeared and qualified aspirants in competitive examination from different States.

Total appeared aspirants = 45000

110. What is the ratio of the number of appeared aspirants from States C and E together to that of the appeared aspirants from States A and F together?

(1) 17:33
(2) 11:13
(3) 13:27
(4) 17:27
(5) None of these

Download Free E-Books for IBPS, Bank Exams

Buy Printed Study Material for IBPS, SBI Exams

IBPS / SBI Special TX: 
General: 
STUDY KITS: 

Pages

Subscribe to RSS - user8's blog